Геометрия 10 класс. Тематические тесты | Тест по геометрии (10 класс) на тему:

1

Точки М, Р, К – середины ребер DA, DB, DC тетраэдра DABC. Назовите прямую, параллельную плоскости FАB.

1) МР             2) РК             3) МК              4)  МК и РК

2

АВСDA1B1C1D1 – прямоугольный параллелепипед. Какая из прямых параллельна плоскости  A1AD?

1) а                   2) b                 3) p                 4)  m 

3

В тетраэдре DАВС AM = MD, AN = NB. Плоскости какой грани параллельна прямая MN?

1) DAB             2) DBC            3) DAC            4)  ABC

4

Выберите верные высказывания:

1) Параллельные прямые не имеют общих точек.

2) Если прямая параллельна данной плоскости, то она параллельна любой прямой, лежащей в этой плоскости.

3) Если прямая параллельна линии пересечения двух плоскостей и не принадлежит ни одной из них, то она параллельна каждой из этих плоскостей.

4)  Существует параллелепипед, у которого все углы граней острые.

Ответ: ______

5

Точки А, В, С и D – середины ребер прямоугольного

параллелепипеда. Назовите параллельные прямые.

1) a || n                           2) a || b               

3) b || c                           4)  a || c

6

Точки А и D – середины ребер параллелепипеда. Выберите верные высказывания:

1) Прямые СD и MN пересекаются.

2) Прямые АВ и MN скрещивающиеся

3) Прямые АВ и СD параллельные.

4) Прямые АВ и MN пересекаются

Ответ: ______

7

Определите взаимное расположение прямых.                

1) a и b – пересекающиеся прямые

2) a и b – параллельные прямые

3) a и b – скрещивающиеся прямые                                

8

Точки А и В – середины ребер параллелепипеда. Определите взаимное расположение прямых.                

1) a и b – пересекающиеся прямые

2) a и b – параллельные прямые

3) a и b – скрещивающиеся прямые                                

9

Два равнобедренных треугольника АВС и АВD с общим основанием АВ расположены так, что точка С не лежит в плоскости АВD. Определите взаимное расположение прямых, содержащих медианы треугольников, проведенных к сторонам ВС и ВD.

1) они параллельны                  2) скрещиваются                   3) пересекаются                                

10

В тетраэдре DАВС АВ = ВС = АС = 10; DA = DB = DC = 20. Через середину ребра ВС плоскость, параллельная АС и ВD. Найдите периметр сечения.

Ответ: ____

Итоговые тесты по геометрии в 10 классе

Параллельность прямых и плоскостей

Вариант 1

1

Точки М, Р, К – середины ребер DA, DB, DC тетраэдра DABC. Назовите прямую, параллельную плоскости FBC.

1) МР 2) РК 3) МК 4) МК и РК

2

АВСDA1B1C1D1 – прямоугольный параллелепипед. Какая из прямых параллельна плоскости A1B1C1?

1) а 2) b 3) p 4) m

3

В тетраэдре DАВС ВК = КС, DP = PC. Плоскости какой грани параллельна прямая РК?

1) DAB 2) DBC 3) DAC 4) ABC

4

Выберите верные высказывания:

1) Две прямые в пространстве называются параллельными, если они не пересекаются.

2) Если одна из двух параллельных прямых параллельна плоскости, то другая прямая либо так же ей параллельна, либо лежит в этой плоскости.

3) Существует такая прямая, которая лежит в плоскости и параллельна прямой, пересекающей данную плоскость.

4) Скрещивающиеся прямые не имеют общих точек.

Ответ: ______

5

Точки А, В, С и D – середины ребер прямоугольного

параллелепипеда. Назовите параллельные прямые.


1) a || n 2) a || b

3) b || c 4) a || c

6

Точки А и D – середины ребер параллелепипеда. Выберите верные высказывания:

1) Прямые СD и MN скрещивающиеся.

2) Прямые АВ и MN лежат в одной плоскости.

3) Прямые СD и MN пересекаются.

4) Прямые АВ и СD скрещивающиеся.

Ответ: ______

7

Определите взаимное расположение прямых.

1) a и bпересекающиеся прямые

2) a и bпараллельные прямые

3) a и bскрещивающиеся прямые

8

Определите взаимное расположение прямых.

1) a и bпересекающиеся прямые

2) a и bпараллельные прямые

3) a и bскрещивающиеся прямые

9

Треугольники АВК и АВF расположены так, что прямые АВ и FK скрещиваются. Как расположены прямые АК и ВF?

1) они параллельны 2) скрещиваются 3) пересекаются

10

В тетраэдре DАВС АВ = ВС = АС = 20; DA = DB = DC = 40. Через середину ребра АС плоскость, параллельная АD и ВC. Найдите периметр сечения.

Ответ: ____

Параллельность прямых и плоскостей

Вариант 2

1

Точки М, Р, К – середины ребер DA, DB, DC тетраэдра DABC. Назовите прямую, параллельную плоскости FАB.

1) МР 2) РК 3) МК 4) МК и РК

2


АВСDA1B1C1D1 – прямоугольный параллелепипед. Какая из прямых параллельна плоскости A1AD?

1) а 2) b 3) p 4) m

3

В тетраэдре DАВС AM = MD, AN = NB. Плоскости какой грани параллельна прямая MN?

1) DAB 2) DBC 3) DAC 4) ABC

4

Выберите верные высказывания:

1) Параллельные прямые не имеют общих точек.

2) Если прямая параллельна данной плоскости, то она параллельна любой прямой, лежащей в этой плоскости.

3) Если прямая параллельна линии пересечения двух плоскостей и не принадлежит ни одной из них, то она параллельна каждой из этих плоскостей.

4) Существует параллелепипед, у которого все углы граней острые.

Ответ: ______

5

Точки А, В, С и D – середины ребер прямоугольного

параллелепипеда. Назовите параллельные прямые.


1) a || n 2) a || b

3) b || c 4) a || c

6

Точки А и D – середины ребер параллелепипеда. Выберите верные высказывания:

1) Прямые СD и MN пересекаются.

2) Прямые АВ и MN скрещивающиеся

3) Прямые АВ и СD параллельные.

4) Прямые АВ и MN пересекаются

Ответ: ______

7


Определите взаимное расположение прямых.

1) a и bпересекающиеся прямые

2) a и bпараллельные прямые

3) a и bскрещивающиеся прямые

8

Точки А и В – середины ребер параллелепипеда. Определите взаимное расположение прямых.

1) a и bпересекающиеся прямые

2) a и bпараллельные прямые

3) a и bскрещивающиеся прямые

9

Два равнобедренных треугольника АВС и АВD с общим основанием АВ расположены так, что точка С не лежит в плоскости АВD. Определите взаимное расположение прямых, содержащих медианы треугольников, проведенных к сторонам ВС и ВD.

1) они параллельны 2) скрещиваются 3) пересекаются

10

В тетраэдре DАВС АВ = ВС = АС = 10; DA = DB = DC = 20. Через середину ребра ВС плоскость, параллельная АС и ВD. Найдите периметр сечения.

Ответ: ____

Контролная работа по геометрии 1 аксиомы располозхение прямых и плоскостей

Контрольная работа по геометрии № 1. 10 класс
Тема: Аксиомы стереометрии, взаимное расположение прямых и плоскости.
1 вариант

1. Точка Р лежит на прямой МN. Назовите плоскость, которой принадлежит точка Р.

1) АВС 2) DBC
3) DAB 4) DAC

2. Каким плоскостям принадлежит точка К?
1) АВС и ABD
2) ABD и BCD
3) ACD и ABD
4) ABC и BCD

3. Выберите верные высказывания:
1) Любые три точки лежат в одной плоскости.
2) Если центр окружности и ее точка лежат в плоскости, то и вся окружность лежит в этой плоскости.
3) Через три точки, лежащих на прямой, проходит только одна плоскость.
4) Через две пересекающихся прямые проходит плоскость , и притом только одна

4. Выберите неверные высказывания:
1) Если три прямые имеют общую точку, то они лежат в одной плоскости.

2) Прямая, пересекающая две стороны треугольника, лежит в плоскости этого треугольника.
3) Две плоскости могут имеет только две общие точки.
4) Три попарно пересекающиеся в разных точках прямые, лежат в одной плоскости.

5. Назовите прямую, по которой пересекаются плоскости (A1BC) и (A1AD).
1) DC 2) A1D1
3) D1D 4) D1C

6. а) Прямые a и b пересекаются. Прямая с скрещивающаяся с прямой а. Могут ли прямые b и c быть параллельными? Ответ обоснуйте.

б) Прямая а параллельна плоскости
·, а прямая b лежит в плоскости
·. Определите, могут ли прямые а и b быть: а) параллельными б) пересекаться в) быть скрещивающимися

7. Плоскость
· проходит через середины боковых сторон АВ и CD трапеции ABCD – точки M и N соответственно.
а) докажите, что AD //
· ; б) Найдите ВС, если AD = 10 см, MN = 8 см.

8. Прямая МА проходит через вершину квадрата ABCD и не лежит в плоскости квадрата.
а) Докажите, что МА и ВС – скрещивающиеся прямые.
б) Найдите угол между прямыми МА и ВС, если угол MAD равен 45 градусов.

9. Две прямые параллельны, если они..
1) не пересекаются 2) перпендикулярны некоторой прямой
3) не пересекаются и лежат в одной плоскости

10. Какое утверждение неверно?
1) две прямые называются параллельными, если они не имеют общих точек

2) две прямые, параллельные третьей прямой, параллельны
3) две прямые, перпендикулярные третьей прямой, параллельны.

11. а) Какое утверждение о прямых верное?
1) 13 EMBED Equation.3 1415
2) 13 EMBED Equation.3 1415
3) 13 EMBED Equation.3 1415
б) укажите прямые скрещивающиеся с прямой АС.

Контрольная работа по геометрии № 1. 10 класс
Тема: Аксиомы стереометрии, взаимное расположение прямых и плоскости.
вариант

1. Точка Р лежит на прямой МN. Назовите плоскость, которой принадлежит точка Р.

1) АВС 2) DBC 3) DAB
· 4) DAC

2. Каким плоскостям принадлежит точка F?

1) АВС и ACD
2) ABD и BCD
3) ACD и BCD
4) ABC и BCD

3. Выберите верные высказывания:
1) Любые четыре точки лежат в одной плоскости.
2) Через прямую и не лежащую на ней точку проходит только одна плоскость.
3) Если три точки окружности лежат в плоскости, то и вся окружность лежит в этой плоскости .
4) Две плоскости могут иметь только одну общую точку.

4. Выберите неверные высказывания:
1) Две окружности, имеющие общий центр, лежат в одной плоскости .
2) Прямая, проходящая через вершину треугольника, лежит в плоскости этого треугольника.
3) Три вершины треугольника принадлежат одной плоскости.

4) Через две параллельные прямые проходит плоскость, и притом только одна.

5. Назовите прямую, по которой пересекаются плоскости DCC1 и A1BC.
1) DC 2) A1D1
3) D1D 4) D1C

6. а) Прямые a и b пересекаются. Прямые а и с параллельны. Могут ли прямые b и с, быть скрещивающимися. Ответ обоснуйте.

б) Прямая а параллельна плоскости
·, а прямая b пересекает плоскость
·. Определите, могут ли прямые а и b быть: а) параллельными б) пересекающимися в) скрещивающимися. Ответ обоснуйте.

7. Плоскость
· проходит через основание трапеции ABCD. Точки М и N — середины боковых сторон трапеции.
а) Докажите, что MN //
·
б) найдите AD, если ВС = 4 см, MN = 6 см.

8. Прямая СD проходит через вершину С треугольника АВС и не лежит в плоскости (АВС). Точки Е и F – середины отрезков АВ и ВС.
а) докажите, что CD и EF – скрещивающиеся прямые
б) найдите угол между прямыми CD и EF, если угол DCA равен 60 градусов.

9. Нельзя провести плоскость через две прямые, если они.
1) параллельные 2) пересекающиеся 3) скрещивающиеся

10. Какое утверждение неверное?
1) если одна из двух параллельных прямых пересекает плоскость, то и другая прямая пересекает эту плоскость
2) если одна из двух параллельных прямых параллельна данной плоскости, то и другая прямая параллельна данной плоскости
3) если две прямые параллельны данной плоскости, то они параллельны друг другу

11. а) Какое утверждение о прямых верное?
1) 13 EMBED Equation.3 1415
2) 13 EMBED Equation.3 1415
3) 13 EMBED Equation.3 1415

б) укажите прямые, скрещивающиеся с прямой А1С1
N

M

D

С

А

В

D1

С1

А1

В1

К

Р

D

С

А

В

D1

С1

А1

В1

найти напряженность магнитного поля

найти напряженность магнитного поля


Задача 70060

Найти напряженность магнитного поля, созданного в точке А проводником с током 10 А, изображенном на рисунке. Отрезки AC = AB = BD = а = 10 см.


Задача 70098

Найти напряженность магнитного поля в центре кругового проволочного витка радиусом 39 см, по которому течет ток 36 А.


Задача 70129

Два круговых витка с одинаковыми токами 61 А, имеющие общий центр, расположены во взаимно перпендикулярных плоскостях. Найти напряженность магнитного поля в центре витков, если радиусы их равны 23 см и 48 см.


Задача 70130

Бесконечно длинный прямой проводник с током 92 А имеет неперекрещивающийся изгиб в виде окружности радиусом 20 см. Найти напряженность магнитного поля в центре этой окружности.


Задача 70139

По двум бесконечно длинным параллельным проводникам, лежащим в одной плоскости, текут противоположно направленные токи 39 А и 23 А. Найти напряженность магнитного поля посередине между проводниками, если расстояние между ними равно 11 см.


Задача 70148

Два круговых витка радиусами 22 см и 57 см с токами 56 А и 43 А лежат в параллельных плоскостях на расстоянии 7 см. Найти напряженность магнитного поля в центре первого витка, если их центры лежат на одном перпендикуляре, а токи направлены в противоположные стороны.


Задача 70170

Длинный проводник с током 50 А изогнут под прямым углом. Найти напряженность магнитного поля на биссектрисе прямого угла на расстоянии 23 см от вершины угла.


Задача 70203

Круговой виток радиусом 18 см с током 6 А расположен в плоскости, параллельной бесконечно длинному проводнику с током 31 А, расположенной от проводника на расстоянии 25 см. Найти напряженность магнитного поля в центре витка, если проводник и диаметр витка лежат в одной плоскости, перпендикулярной плоскости витка.


Задача 70207

На деревянный цилиндр, имеющий основание радиусом 31 см и высоту 27 см, намотаны два взаимно перпендикулярных витка с одинаковыми токами 78 А. Витки проходят через центры оснований. Найти напряженность магнитного поля в середине оси цилиндра.


Задача 70285

На бесконечно длинном проводе образована круговая петля, касательная к проводу. Плоскость петли повернули перпендикулярно проводу. Найдите напряженность магнитного поля в центре петли, если ее радиус R = 10,0 см, а сила тока в проводнике I = 15,0 A.


Задача 15355

Два круговых витка радиусом R = 4 см каждый расположены в параллельных плоскостях на расстоянии d = 10 см друг от друга. По виткам текут токи I1 = I2 = 2 А. Найти напряженность H магнитного поля на оси витков в точке, находящейся на равном расстоянии от них. Задачу решить, когда: а) токи в витках текут в одном направлении; б) токи в витках текут в противоположных направлениях.


Задача 15356

Два круговых витка радиусом R = 4 см каждый расположены в параллельных плоскостях на расстоянии d = 5 см друг от друга. По виткам текут токи I1 = I2 = 4 A, Найти напряженность H магнитного поля в центре одного из витков. Задачу решить, когда: а) токи в витках текут в одном направлении; б) токи в витках текут в противоположных направлениях.


Задача 15358

Два круговых витка расположены в двух взаимно перпендикулярных плоскостях так, что центры этих витков совпадают. Радиус каждого витка R = 2 см, токи в витках I1 = I2 = 5 А. Найти напряженность Н магнитного поля в центре этих витков.


Задача 15359

Из проволоки длиной l = 1 м и сделана квадратная рамка. По рамке течет ток I = 10 А. Найти напряженность H магнитного поля в центре рамки.


Задача 12802

Два круговых витка с одинаковыми токами 64 А, имеющие общий центр, расположены во взаимно перпендикулярных плоскостях. Найти напряжённость магнитного поля в центре витков, если радиусы их равны 38 см и 17 см.


Задача 12803

Два круговых витка с одинаковыми токами 27 А, имеющие общий центр, расположены во взаимно перпендикулярных плоскостях. Найти напряженность магнитного поля в центре витков, если радиусы их равны 41 и 26 см.


Задача 12804

Два круговых витка с одинаковыми токами 2 А имеют общий центр и расположены во взаимно перпендикулярных плоскостях. Найти напряженность магнитного поля в центре витков, если радиусы их равны 4 см и 2 см.


Задача 12883

По двум бесконечно длинным параллельным проводникам, лежащим в одной плоскости, текут противоположно направленные токи 33 А и 13 А. Найти напряжённость магнитного поля посередине между проводниками, если расстояние между ними равно 27 см.


Задача 12894

Бесконечно длинный проводник с током 12 А изогнут под прямым углом. Найти напряжённость магнитного поля на биссектрисе прямого угла на расстоянии 96 см от вершины угла.


Задача 12985

Бесконечно длинный проводник с током 41 А изогнут под прямым углом. Найти напряжённость магнитного поля на биссектрисе прямого угла на расстоянии 87 см от вершины угла.


Задача 12967

Два круговых витка с одинаковыми токами 59 А, имеющие общий центр, расположены во взаимно перпендикулярных плоскостях. Найти напряженность магнитного поля в центре витков, если радиусы их равны 27 см и 32 см.


Задача 12978

Бесконечно длинный проводник с током 21 А изогнут под прямым углом. Найти напряженность магнитного поля на биссектрисе прямого угла на расстоянии 97 см от вершины угла.


Задача 13696

Два круговых витка с одинаковыми токами 98 А, имеющие общий центр, расположены во взаимно перпендикулярных плоскостях. Найти напряженность магнитного поля в центре витков, если радиусы их равны 25 см и 37 см.


Задача 17681

Три проводника с токами I, I/4, 3I/4 лежат в одной плоскости и соединены в точке О. Найти напряженность магнитного поля на прямой, проходящей через точку О, перпендикулярной всем трем проводникам. (I = 10 А, расстояние до точки наблюдения 5 см).


Задача 17699

Два круговых витка радиусом 4 см каждый расположены в параллельных плоскостях на расстояния 3 см друг от друга (расстояние между центрами витков). По виткам течет ток 4 А в каждом. Найти напряженность магнитного поля в центре одного из витков. Рассмотреть случаи: 1) направление токов совпадают, 2) токи противоположного направления.


Задача 20104

Два электрона движутся в одном направлении вдоль одной прямой с одинаковой по модулю скоростью 104 м/с. Найти напряженность магнитного поля зарядов при расстоянии между ними 4·10–8 см. Точка, для которой определяется напряженность магнитного поля, лежит на серединном к траектории перпендикуляре на высоте 3·10–8 см.


Задача 20507

Найти напряженность магнитного поля в центре дуги радиусом 0,2 м, замыкающей два параллельных полубесконечных проводника, если они лежат в плоскости, перпендикулярной плоскости дуги, а сила тока в цепи 14 А.


Задача 20508

Бесконечный прямой провод на некотором участке содержит петлю в виде квадрата со стороной 12 см, одна из диагоналей которого перпендикулярна прямолинейному участку, касающемуся вершины, причем петля и провод лежат в одной плоскости. Найти напряженность магнитного поля в центре такой петли, если ток в проводнике равен 24 А.


Задача 21504

По проводу, согнутому в виде правильного шестиугольника с длиной стороны 10 см течет ток 5 А. Найти напряженность магнитного поля в центре шестиугольника.


Окружность

Окружность — геометрическая фигура, состоящая из всех точек плоскости, расположенных на заданном расстоянии от данной точки.

Данная точка (O) называется центром окружности.
Радиус окружности — это отрезок, соединяющий центр с какой-либо точкой окружности. Все радиусы имеют одну и ту же длину (по определению).
Хорда — отрезок, соединяющий две точки окружности. Хорда, проходящая через центр окружности, называется диаметром. Центр окружности является серединой любого диаметра.
Любые две точки окружности делят ее на две части. Каждая из этих частей называется дугой окружности. Дуга называется полуокружностью, если отрезок, соединяющий её концы, является диаметром.
Длина единичной полуокружности обозначается через π.
Сумма градусных мер двух дуг окружности с общими концами равна 360º.
Часть плоскости, ограниченная окружностью, называется кругом.
Круговой сектор — часть круга, ограниченная дугой и двумя радиусами, соединяющими концы дуги с центром круга. Дуга, которая ограничивает сектор, называется дугой сектора.
Две окружности, имеющие общий центр, называются концентрическими.
Две окружности, пересекающиеся под прямым углом, называются ортогональными.

Взаимное расположение прямой и окружности

  1. Если расстояние от центра окружности до прямой меньше радиуса окружности (d ), то прямая и окружность имеют две общие точки. В этом случае прямая называется секущей по отношению к окружности.
  2. Если расстояние от центра окружности до прямой равно радиусу окружности, то прямая и окружность имеют только одну общую точку. Такая прямая называется касательной к окружности, а их общая точка называется точкой касания прямой и окружности.
  3. Если расстояние от центра окружности до прямой больше радиуса окружности, то прямая и окружность не имеют общих точек
  4. .

Центральные и вписанные углы

Центральный угол — это угол с вершиной в центре окружности.
Вписанный угол — угол, вершина которого лежит на окружности, а стороны пересекают окружность.

Теорема о вписанном угле

Вписанный угол измеряется половиной дуги, на которую он опирается.

  • Следствие 1.
    Вписанные углы, опирающиеся на одну и ту же дугу, равны.

  • Следствие 2.
    Вписанный угол, опирающийся на полуокружность — прямой.

Теорема о произведении отрезков пересекающихся хорд.

Если две хорды окружности пересекаются, то произведение отрезков одной хорды равно произведению отрезков другой хорды.

Основные формулы

  • Длина окружности:
C = 2∙π∙R
  • Длина дуги окружности:
R = С/(2∙π) = D/2 D = C/π = 2∙R
  • Длина дуги окружности:
l = (π∙R) / 180∙α,
где α — градусная мера длины дуги окружности)
  • Площадь круга:
S = π∙R2
  • Площадь кругового сектора:
S = ((π∙R2) / 360)∙α

Уравнение окружности

  • В прямоугольной системе координат уравнение окружности радиуса r с центром в точке C (xо;yо) имеет вид:
(x — xо)2 + (y — yо)2 = r2
  • Уравнение окружности радиуса r с центром в начале координат имеет вид:
x2 + y2 = r2

Другие заметки по алгебре и геометрии

Т 3 перпендикулярность в пространстве вариант 1. Перпендикулярность прямых в пространстве. Визуальный гид (2019). Координаты точки основания перпендикуляра к прямой

«Перпендикулярные прямые в пространстве.

Перпендикулярность прямой и плоскости»

Вариант 1

Уровень А

1. Какое утверждение верно?

1) Если одна из двух прямых перпендикулярна к третьей прямой, то и другая прямая перпендикулярна к этой прямой.

2) Если две прямые перпендикулярны к третьей прямой, то они параллельны.

3) Если две прямые перпендикулярны к плоскости, то они параллельны.

2. ABCD – прямоугольник, BM ┴ (ABC ) . Тогда неверно, что…

1) BM AC ;

2) AM AD ;

3) MD DC .

3. Прямая m перпендикулярна к прямым a и b , лежащим в плоскости α, но m не перпендикулярна к плоскости α. Тогда прямые a и b

1) параллельны;

2) пересекаются;

3) скрещиваются.

4. Плоскость α проходит через вершину А ромба ABCD перпендикулярно диагонали АС. Тогда диагональ BD …

1) перпендикулярна плоскости α;

2) параллельна плоскости α;

3) лежит в плоскости α.

5. a α , b ┴ α. Тогда прямые a и b не могут быть …

1) скрещивающимися;

2) перпендикулярными;

3) параллельными.

6. ABCD – параллелограмм, BD α, AC ┴ α. Тогда ABCD не может быть …

1) прямоугольником;

2) квадратом;

3) ромбом.

1) радиусам; 2) диаметрам; 3) хордам.

8. Какое утверждение верно:

1) Прямая и не проходящая через неё плоскость, перпендикулярные другой плоскости, параллельны между собой.

2) Плоскость и перпендикулярная данной плоскости, перпендикулярна и к прямой, параллельной данной плоскости.

3) Плоскость, перпендикулярная данной прямой, перпендикулярна и к плоскости, параллельной данной прямой.

9. AC ┴ (BDM ) . Тогда отрезок BM в треугольнике АВС является …

1) медианой;

2) высотой;

3) биссектрисой.

Вариант 1

https://pandia.ru/text/78/082/images/image006_123.gif»>(а, ВМ ) = …

https://pandia.ru/text/78/082/images/image003_184.gif»> α , СМ = МВ, АМ = 2,5 см, АС = 3 см. Тогда АВ = …

https://pandia.ru/text/78/082/images/image009_91.gif»>см. АС BD = О . FO ┴ (ABC ), FO = см. Расстояние от точки F до вершины квадрата равно …

https://pandia.ru/text/78/082/images/image013_21.jpg» align=»left»>

5. ABCD – прямоугольник. BF ┴ (ABC ). CF = 20 см, DF = 25 см. Тогда длина отрезка CD равна …

https://pandia.ru/text/78/082/images/image015_17.jpg» align=»left»>лежит в плоскости α .

5. ABCD — параллелограмм, АВhttps://pandia.ru/text/78/082/images/image016_17.jpg» align=»left»>скрещивающимися.

7. Dhttps://pandia.ru/text/78/082/images/image006_123.gif»> (АВ, CD) =600.

8. Какое утверждение неверное?

1) Через любую точку пространства проходит прямая, перпендикулярная к данной плоскости, и притом только одна.

2) Через точку, не лежащую на данной прямой, можно построить только одну плоскость, перпендикулярную данной прямой.

3) Через точку, не лежащую на данной прямой, можно построить только одну прямую, перпендикулярную данной прямой.

1. Найдите угол между пересекающимися диагоналями граней куба.

2. В кубе A…D 1 найдите угол между прямыми AD 1 и CB 1 .

3. Диагональ прямоугольного параллелепипеда, основанием которого является квадрат, в два раза больше стороны основания. Найдите углы между диагоналями параллелепипеда, которые лежат в одном диагональном сечении.

1) 45 0 и 45 0 .

2) 90 0 и 90 0 .

3) 30 0 и 60 0 .

4) 60 0 и 120 0 .

4. Диагональ прямоугольного параллелепипеда, основанием которого является квадрат, в два раза больше стороны основания. Найдите углы между диагоналями параллелепипеда, которые лежат в разных диагональных сечениях.

1) 45 0 и 135 0 .

2) 90 0 и 90 0 .

3) 30 0 и 150 0 .

4) 60 0 и 120 0 .

5. Найдите угол между скрещивающимися ребрами правильной треугольной пирамиды.

6. Из точки, не принадлежащей плоскости опущен на нее перпендикуляр и проведена наклонная. Найдите проекцию наклонной, если перпендикуляр равен 12 см, а наклонная 15 см.

7. Найдите геометрическое место прямых, перпендикулярных данной прямой и проходящих через данную на ней точку.

2) Плоскость, перпендикулярная данной прямой.

3) Плоскость, параллельная данной прямой.

4) Плоскость, перпендикулярная данной прямой и проходящая через данную точку.

8. Найдите геометрическое место точек, равноудаленных от двух данных точек.

1) Перпендикуляр, проведенный к середине отрезка, соединяющего данные точки.

3) Плоскость, перпендикулярная прямой, проходящей через данные точки.

4) Плоскость, перпендикулярная отрезку, соединяющему данные точки и проходящая через его середину.

9. Из данной точки к плоскости проведены перпендикуляр и наклонная. Зная, что их разность равна 25 см, а расстояние между их серединами 32,5 см, найдите наклонную.

10. Концы отрезка находятся от данной плоскости на расстоянии 26 см и 37 см. Его ортогональная проекция на плоскость равна 6 дм. Найдите отрезок.

11. Один из катетов прямоугольного равнобедренного треугольника лежит в плоскости, а другой наклонен к ней под углом 45 0 . Найдите угол между гипотенузой этого треугольника и данной плоскостью.

12. Найдите угол наклона отрезка к плоскости, если его ортогональная проекция на эту плоскость в два раза меньше самого отрезка.

13. Найдите геометрическое место точек, равноудаленных от всех точек окружности.

1) Центр окружности.

2) Окружность.

3) Плоскость, перпендикулярная плоскости окружности и проходящая через ее центр.

14. Найдите геометрическое место точек, равноудаленных от всех сторон ромба.

1) Перпендикуляр, проведенный к плоскости ромба и проходящий через его вершину.

2) Плоскость, перпендикулярная к плоскости ромба и проходящая через его диагональ.

3) Перпендикуляр, проведенный к плоскости ромба и проходящий через точку пересечения его диагоналей.

4) Окружность, вписанная в ромб.

15. Найдите высоту правильной треугольной пирамиды, если сторона ее основания равна a , боковое ребро b .

3) .

16. Найдите двугранный угол j между боковыми гранями правильной четырехугольной пирамиды, все ребра которой равны 1.

17. Точка A находится от одной из двух перпендикулярных плоскостей на расстоянии 4 см, а от другой на 16 см. Найдите расстояние от точки A до линии пересечения плоскостей.

18. Найдите двугранный угол при основании правильной четырехугольной пирамиды, если ее высота равна 2 см, а сторона основания 4 см.

19. Точка B , удаленная от ребра двугранного угла на расстояние a , отстоит от каждой его грани на одинаковое расстояние. Найдите это расстояние, если двугранный угол равен j.

1) a sinj.

2) a cosj.

3) a sin .

4) a cos .

20. Точка E принадлежит плоскости a, точка F принадлежит плоскости b. Плоскости перпендикулярны. Ортогональные проекции отрезка EF , равного 10 см, на плоскости a и b соответственно равны 8 см и 7,5 см. Найдите проекцию отрезка EF на линию пересечения плоскостей a и a.

ОТВЕТЫ

Номер задания Номер теста
4) 3) 3) 4) 4) 2) 1)
4) 3) 4) 3) 3) 1) 2)
2) 4) 2) 3) 4) 1) 4)
4) 1) 4) 3) 2) 3) 3)
2) 1) 4) 3) 3) 4) 3)
2) 2) 2) 2) 3) 4) 3)
4) 3) 4) 2) 1) 4) 4)
4) 2) 4) 2) 2) 3) 2)
3) 3) 3) 1) 4) 3) 3)
1) 4) 1) 4) 3) 3) 4)
3) 1) 2) 2) 2) 3) 3)
2) 2) 3) 3) 1) 2) 1)
2) 3) 4) 4) 4) 4) 3)
4) 4) 3) 3) 2) 3) 4)
3) 4) 3) 2) 1) 2) 4)
3) 2) 2) 2) 4) 3) 3)
3) 4) 4) 2) 2) 2) 4)
4) 3) 2) 4) 3) 2) 2)
2) 4) 3) 1) 3) 2) 2)
1) 2) 1) 4) 2) 3) 4)

Две прямые в пространстве называются перпендикулярными, если угол между ними равен 90 o .


рис. 37
Перпендикулярные прямые могут пересекаться и могут быть скрещивающимися.

Лемма. Если одна из двух параллельных прямых перпендикулярна к третьей прямой, то и другая прямая перпендикулярна к этой прямой.

Определение. Прямая называется перпендикулярной к плоскости, если она перпендикулярна к любой прямой, лежащей в плоскости.

Говорят также, что плоскость перпендикулярна к прямой а.


рис. 38
Если прямая а перпендикулярна к плоскости , то она, очевидно, пересекает эту плоскость. В самом деле, если бы прямая а не пересекала плоскость , то она лежала бы в этой плоскости или была бы параллельна ей.

Но в том и в другом случае в плоскости имелись бы прямые, не перпендикулярные к прямой а, например прямые, параллельные ей, что невозможно. Значит, прямая а пересекает плоскость .

Связь между параллельностью прямых и их перпендикулярностью к плоскости.

Признак перпендикулярности прямой и плоскости.

Замечания.

  1. Через любую точку пространства проходит плоскость, перпендикулярная к данной прямой, и притом единственная.
  2. Через любую точку пространства проходит прямая, перпендикулярная к данной плоскости, и притом только одна.
  3. Если две плоскости перпендикулярны к прямой, то они параллельны.
Задачи и тесты по теме «Тема 5. «Перпендикулярность прямой и плоскости».»
  • Перпендикулярность прямой и плоскости
  • Двугранный угол. Перпендикулярность плоскостей — Перпендикулярность прямых и плоскостей 10 класс

    Уроков: 1 Заданий: 10 Тестов: 1

  • Перпендикуляр и наклонные. Угол между прямой и плоскостью — Перпендикулярность прямых и плоскостей 10 класс

    Уроков: 2 Заданий: 10 Тестов: 1

  • Параллельность прямых, прямой и плоскости

    Уроков: 1 Заданий: 9 Тестов: 1

  • Параллельность плоскостей — Параллельность прямых и плоскостей 10 класс

    Уроков: 1 Заданий: 8 Тестов: 1

Материал темы обобщает и систематизирует известные Вам из планиметрии сведения о перпендикулярности прямых. Изучение теорем о взаимосвязи параллельности и перпендикулярности прямых и плоскостей в пространстве, а также материал о перпендикуляре и наклонных целесообразно сочетать с систематическим повторением соответствующего материала из планиметрии.

Решения практически всех задач на вычисление сводятся к применению теоремы Пифагора и следствий из нее. Во многих задачах возможность применения теоремы Пифагора или следствий из нее обосновывается теоремой о трех перпендикулярах или свойствами параллельности и перпендикулярности плоскостей.

13.11.2016 14:35

Тестовые задания по геометрии к разделу «Прямые и плоскости в пространстве»1.Аксиомы стереометрии. 2.Параллельность прямых и плоскостей. 3.Перпендикулярность прямых и плоскостей. Ответы в конце разработки

Просмотр содержимого документа


«Тестовые задания по геометрии к разделу «Прямые и плоскости в пространстве» 1 курс СПО»

Раздел № 3.

Прямые и плоскости в пространстве

Предмет стереометрии. Основные понятия и аксиомы стереометрии.

Пространственные фигуры.

Параллельность прямыхв пространстве. Параллельность двух плоскостей.

Векторы в пространстве.

Параллельный перенос.

Сечение многогранников.

Перпендикулярность прямых, прямой и плоскости.

Перпендикуляр и наклонная.

Угол между прямой и плоскостью.

Двугранный угол. Перпендикулярность плоскостей.

Аксиомы стереометрии

Вариант 1

1) АВС 2) DBC 3) DAB 4) DAC

Каким плоскостям принадлежит точка К?

1) АВС и ABD

Выберите верные высказывания:

1) Любые три точки лежат в одной плоскости.

2) Если центр окружности и ее точка лежат в плоскости, то и вся окружность лежит в этой плоскости.

3) Через три точки, лежащих на прямой, проходит только одна плоскость.

4) Через две пересекающихся прямые проходит плоскость, и притом только одна.

Ответ: ______

Выберите неверные высказывания:

1) Если три прямые имеют общую точку, то они лежат в одной плоскости.

3) Две плоскости могут имеет только две общие точки.

4) Три попарно пересекающиеся в разных точках прямые, лежат в одной плоскости.

Ответ: ______

Назовите прямую, по которой пересекаются плоскости A 1 BC и A 1 AD.

1) DC 2) A 1 D 1

3) D 1 D 4) D 1 C

Назовите прямую, по которой пересекаются плоскости DCC 1 и A 1 AD.

1) DC 2) A 1 D 1

3) D 1 D 4) D 1 C

Прямые АВ и CD пересекаются. Через прямую АВ проведена плоскость. Назовите линию пересечения данной плоскости с плоскостью ВСD.

1) АС 2) АB 3) BС 4) ВD

Прямые АВ и CD пересекаются. Через точки В и D проведена плоскость. Назовите линию пересечения данной плоскости с плоскостью AСD.

1) АС 2) АB 3) BС 4) ВD

Вариант 2

Точка Р лежит на прямой МN. Назовите плоскость, которой принадлежит точка Р.

1) АВС 2) DBC 3) DAB 4) DAC

Каким плоскостям принадлежит точка F?

1) АВС и ACD

Выберите верные высказывания:

1) Любые четыре точки лежат в одной плоскости.

2) Через прямую и не лежащую на ней точку проходит только одна плоскость.

3) Если три точки окружности лежат в плоскости, то и вся окружность лежит в этой плоскости.

4) Две плоскости могут иметь только одну общую точку.

Ответ: ______

Выберите неверные высказывания:

1) Две окружности, имеющие общий центр, лежат в одной плоскости.

3) Три вершины треугольника принадлежат одной плоскости.

4) Через две параллельные прямые проходит плоскость, и притом только одна.

Ответ: ______

Назовите прямую, по которой пересекаются плоскости DCC 1 и A 1 BC.

1) DC 2) A 1 D 1

3) D 1 D 4) D 1 C

Назовите прямую, по которой пересекаются плоскости ABC и C 1 CB.

1) BC 2) B 1 C 1

3) A 1 B 4) B 1 B

Прямые АВ и CD пересекаются. Через прямую CD проведена плоскость. Назовите линию пересечения данной плоскости с плоскостью AВС.

1) СD 2) АD 3) BС 4) ВD

Прямые АВ и CD пересекаются. Через точки A и D проведена плоскость. Назовите линию пересечения данной плоскости с плоскостью BСD.

1) АС 2) АD 3) BС 4) ВD

Вариант 1

Точки М, Р, К – середины ребер DA, DB, DC тетраэдра DABC. Назовите прямую, параллельную плоскости FBC.

1) МР 2) РК 3) МК 4) МК и РК

АВСDA 1 B 1 C 1 D 1 – прямоугольный параллелепипед. Какая из прямых параллельна плоскости A 1 B 1 C 1 ?

1) а 2) b 3) p 4) m

В тетраэдре DАВС ВК = КС, DP = PC. Плоскости какой грани параллельна прямая РК?

1) DAB 2) DBC 3) DAC 4) ABC

Выберите верные высказывания:

1) Две прямые в пространстве называются параллельными, если они не пересекаются.

2) Если одна из двух параллельных прямых параллельна плоскости, то другая прямая либо так же ей параллельна, либо лежит в этой плоскости.

3) Существует такая прямая, которая лежит в плоскости и параллельна прямой, пересекающей данную плоскость.

4) Скрещивающиеся прямые не имеют общих точек.

Ответ: ______


1) a || n 2) a || b

3) b || c 4) a || c

верные высказывания:

1) Прямые СD и MN скрещивающиеся.

2) Прямые АВ и MN лежат в одной плоскости.

3) Прямые СD и MN пересекаются.

4) Прямые АВ и СD скрещивающиеся.

Ответ: ______


1) a и b пересекающиеся прямые

2) a и b параллельные прямые

3) a и b скрещивающиеся прямые

Определите взаимное расположение прямых.

1) a и b пересекающиеся прямые

2) a и b параллельные прямые

3) a и b скрещивающиеся прямые

Треугольники АВК и АВF расположены так, что прямые АВ и FK скрещиваются. Как расположены прямые АК и ВF?

В тетраэдре DАВС АВ = ВС = АС = 20; DA = DB = DC = 40. Через середину ребра АС плоскость, параллельная АD и ВC. Найдите периметр сечения.

Ответ: ____

Параллельность прямых и плоскостей

Вариант 2

Точки М, Р, К – середины ребер DA, DB, DC тетраэдра DABC. Назовите прямую, параллельную плоскости FАB.

1) МР 2) РК 3) МК 4) МК и РК


АВСDA 1 B 1 C 1 D 1 – прямоугольный параллелепипед. Какая из прямых параллельна плоскости A 1 AD?

1) а 2) b 3) p 4) m

В тетраэдре DАВС AM = MD, AN = NB. Плоскости какой грани параллельна прямая MN?

1) DAB 2) DBC 3) DAC 4) ABC

Выберите верные высказывания:

1) Параллельные прямые не имеют общих точек.

2) Если прямая параллельна данной плоскости, то она параллельна любой прямой, лежащей в этой плоскости.

3) Если прямая параллельна линии пересечения двух плоскостей и не принадлежит ни одной из них, то она параллельна каждой из этих плоскостей.

4) Существует параллелепипед, у которого все углы граней острые.

Ответ: ______

Точки А, В, С и D – середины ребер прямоугольного

параллелепипеда. Назовите параллельные прямые.


1) a || n 2) a || b

3) b || c 4) a || c

Точки А и D – середины ребер параллелепипеда. Выберите верные высказывания:

1) Прямые СD и MN пересекаются.

2) Прямые АВ и MN скрещивающиеся

3) Прямые АВ и СD параллельные.

4) Прямые АВ и MN пересекаются

Ответ: ______


Определите взаимное расположение прямых.

1) a и b пересекающиеся прямые

2) a и b параллельные прямые

3) a и b скрещивающиеся прямые

Точки А и В – середины ребер параллелепипеда. Определите взаимное расположение прямых.

1) a и b пересекающиеся прямые

2) a и b параллельные прямые

3) a и b скрещивающиеся прямые

Два равнобедренных треугольника АВС и АВD с общим основанием АВ расположены так, что точка С не лежит в плоскости АВD. Определите взаимное расположение прямых, содержащих медианы треугольников, проведенных к сторонам ВС и ВD.

1) они параллельны 2) скрещиваются 3) пересекаются

В тетраэдре DАВС АВ = ВС = АС = 10; DA = DB = DC = 20. Через середину ребра ВС плоскость, параллельная АС и ВD. Найдите периметр сечения.

Ответ: ____

Вариант 1

Через сторону АВ треугольника АВС проведена плоскость, перпендикулярная к стороне ВС. Определите вид треугольника относительно углов.

Треугольник АВС – правильный, О – центр треугольника. Расстояние от точки М до вершины А равно 3. Найдите высоту треугольника.

Ответ: ____

АВСD – параллелограмм; Найдите периметр параллелограмма.

1) 20 2) 25 3) 40 4) 60

Через вершину А треугольника ABC проведена плос­кость α, параллельная ВС. Расстояние от ВС до плоскости α равно 12. Найдите расстояние от точки пересечения ме­диан треугольника АВС до этой плоскости.

1) 8 2) 6 3) 12 4) 18

Высота ромба равна 12. Точка М равноудалена от всех сторон ромба и находится на расстоянии, равном 8, от его плоскости. Чему равно расстояние точки М до сторон ромба?

Ответ: ____

Выберите верные высказывания:

2) Две прямые, перпендикулярные к одной плоскости, параллельны.

3) Длина перпендикуляра меньше длины наклонной, проведенной из той же точки.

4) Две скрещивающиеся прямые могут быть перпендикулярными к одной плоскости.

Ответ: ______

Отрезок АВ упирается концами А и В в грани прямого двугранного угла. Расстояния от точек А и В до ребра равны 1, а длина отрезка АВ равна 3. Найдите длину про­екции этого отрезка на ребро.

В тетраэдре DABC АО пресекает ВС в точке Е; Найдите.

Прямоугольник ABCD и параллелограмм ВЕМС распо­ложены так, что их плоскости взаимно перпендикулярны. Найдите угол MCD.

Перпендикулярность прямых и плоскостей

Вариант 2

Через сторону АD параллелограмма АВСD, проведена плоскость, перпендикулярная к стороне DС. Определите вид треугольника АВС.

1) остроугольный 2) прямоугольный 3) тупоугольный

Треугольник АВС – правильный, О – центр треугольника. Высота треугольника равна 3. Найдите расстояние от точки М до вершин треугольника.

Ответ: ____

АВСD – параллелограмм; Найдите BD.

1) 20 2) 15 3) 40 4) 10

Через вершину А треугольника ABC проведена плос­кость α, параллельная ВС. Расстояние от точки пересече­ния медиан треугольника АВС до этой плоскости равно 4. На каком расстоянии от плоскости находится ВС?

1) 8 2) 6 3) 12 4) 14

Точка Р удалена от всех сторон ромба на расстояние» равное, и находится от его плоскости на расстоянии равном 2. Чему равна сторона ромба, если его угол 30°?

Ответ: ____

На рисунке Найдите угол между МС и плоскостью АМВ.

1) 30 0 2) 60 0 3) 90 0 4) 45 0

Выберите верные высказывания:

1) Угол между прямой и плоскостью может быть не больше 90 0 .

2) Две плоскости, перпендикулярные к одной прямой, пересекаются.

3) Длина перпендикуляра больше длины наклонной, проведенной из той же точки.

4) Диагональ прямоугольного параллелепипеда больше любого из ребер.

Ответ: ______

Отрезок АВ упирается концами А и В в грани прямого двугранного угла. Расстояния от точек А и В до ребра равны 2, а длина отрезка АВ равна 4. Найдите длину про­екции этого отрезка на ребро.

В тетраэдре DABC основание ABC — правильный тре­угольник. Вершина D проецируется в его центр О. Найди­те угол между плоскостью ADO и гранью DCB.

1) 30 0 2) 60 0 3) 90 0 4) 45 0

Треугольник АМВ и прямоугольник ABCD расположе­ны так, что их плоскости взаимно перпендикулярны. Най­дите угол MAD.

1) 90 0 2) 60 0 3) 30 0 4) 45 0

Тест 1

Вариант 1

Вариант 2

Тест 2

Вариант 1

Вариант 2

Тест 3

Вариант 1

Вариант 2

Например, перпендикулярность прямых m {\displaystyle m} и n {\displaystyle n} записывают как m ⊥ n {\displaystyle m\perp n} .

Энциклопедичный YouTube

    1 / 5

    ✪ 10 класс, 17 урок, Признак перпендикулярности прямой и плоскости

    ✪ стереометрия ПАРАЛЛЕЛЬНЫЕ ПРЯМЫЕ перпендикулярные к плоскости

    ✪ Перпендикулярность прямой и плоскости. Геометрия 10-11 классы. Урок 7

    ✪ стереометрия ПРИЗНАК ПЕРПЕНДИКУЛЯРНОСТИ ПРЯМОЙ и ПЛОСКОСТИ

    ✪ 10 класс, 15 урок, Перпендикулярные прямые в пространстве

    Субтитры

На плоскости

Перпендикулярные прямые на плоскости

В аналитическом выражении прямые, заданные линейными функциями y = tg ⁡ α 1 x + b 1 {\displaystyle y=\operatorname {tg} \alpha _{1}x+b_{1}} и y = tg ⁡ α 2 x + b 2 {\displaystyle y=\operatorname {tg} \alpha _{2}x+b_{2}} будут перпендикулярны, если выполнено условие α 2 = 1 2 π + α 1 {\displaystyle \alpha _{2}={\frac {1}{2}}\pi +\alpha _{1}} . Эти же прямые будут перпендикулярны, если tg ⁡ α 1 tg ⁡ α 2 = − 1 {\displaystyle \operatorname {tg} \alpha _{1}\operatorname {tg} \alpha _{2}=-1} . (Здесь α 1 , α 2 {\displaystyle \alpha _{1},\alpha _{2}} — углы наклона прямой к горизонтали)

Построение перпендикуляра

Шаг 1: (красный ) С помощью циркуля проведём полуокружность с центром в точке P, получив точки А» и В».

Шаг 2: (зелёный ) Не меняя радиуса, построим две полуокружности с центром в точках A» и В» соответственно, проходящими через точку Р. Кроме точки Р есть ещё одна точка пересечения этих полуокружностей, назовём её Q.

Шаг 3: (синий ) Соединяем точки Р и Q. PQ и есть перпендикуляр к прямой АВ.

Координаты точки основания перпендикуляра к прямой

A (x a , y a) {\displaystyle A(x_{a},y_{a})} и B (x b , y b) {\displaystyle B(x_{b},y_{b})} — прямая, O (x o , y o) {\displaystyle O(x_{o},y_{o})} — основание перпендикуляра, опущенного из точки P (x p , y p) {\displaystyle P(x_{p},y_{p})} .{2}}}} ; y o = (x b − x a) ⋅ (x p − x o) (y b − y a) + y p {\displaystyle y_{o}={\frac {(x_{b}-x_{a})\cdot (x_{p}-x_{o})}{(y_{b}-y_{a})}}+y_{p}} .

В трёхмерном пространстве

Перпендикулярные прямые

Две прямые в пространстве перпендикулярны друг другу, если они соответственно параллельны некоторым двум другим взаимно перпендикулярным прямым, лежащим в одной плоскости. Две прямые, лежащие в одной плоскости, называются перпендикулярными (или взаимно перпендикулярными), если они образуют четыре прямых угла.

Перпендикулярность прямой к плоскости

Определение : Прямая называется перпендикулярной к плоскости, если она перпендикулярна всем прямым, лежащим в этой плоскости.

Признак : Если прямая перпендикулярна каждой из двух пересекающихся прямых плоскости, то она перпендикулярна этой плоскости.

Плоскость , перпендикулярная одной из двух параллельных прямых, перпендикулярна и другой. Через любую точку пространства проходит прямая, перпендикулярная к данной плоскости, и притом только одна.

Перпендикулярные плоскости

Две плоскости называются перпендикулярными, если двугранный угол между ними равен 90°.

В многомерных пространствах

Перпендикулярность плоскостей в 4-мерном пространстве

Перпендикулярность плоскостей в четырёхмерном пространстве имеет два смысла: плоскости могут быть перпендикулярны в 3-мерном смысле, если они пересекаются по прямой (а следовательно, лежат в одной гиперплоскости), и двугранный угол между ними равен 90°.

Плоскости могут быть также перпендикулярны в 4-мерном смысле, если они пересекаются в точке (а следовательно, не лежат в одной гиперплоскости), и любые 2 прямые, проведённые в этих плоскостях через точку их пересечения (каждая прямая в своей плоскости), перпендикулярны.

В 4-мерном пространстве через данную точку можно провести ровно 2 взаимно перпендикулярные плоскости в 4-мерном смысле (поэтому 4-мерное евклидово пространство можно представить как декартово произведение двух плоскостей).{k}} , то есть (∀ a → ∈ L 1) (∀ b → ∈ L k) a → b → = 0 {\displaystyle (\forall {\vec {a}}\in L_{1})\ (\forall {\vec {b}}\in L_{k})\ {\vec {a}}{\vec {b}}=0}

Перпендикулярность в пространстве вариант 2. Тест «Перпендикулярные прямые в пространстве. Перпендикулярность прямой и плоскости. Параллельность прямых и плоскостей

Например, перпендикулярность прямых m {\displaystyle m} и n {\displaystyle n} записывают как m ⊥ n {\displaystyle m\perp n} .

Энциклопедичный YouTube

    1 / 5

    ✪ 10 класс, 17 урок, Признак перпендикулярности прямой и плоскости

    ✪ стереометрия ПАРАЛЛЕЛЬНЫЕ ПРЯМЫЕ перпендикулярные к плоскости

    ✪ Перпендикулярность прямой и плоскости. Геометрия 10-11 классы. Урок 7

    ✪ стереометрия ПРИЗНАК ПЕРПЕНДИКУЛЯРНОСТИ ПРЯМОЙ и ПЛОСКОСТИ

    ✪ 10 класс, 15 урок, Перпендикулярные прямые в пространстве

    Субтитры

На плоскости

Перпендикулярные прямые на плоскости

В аналитическом выражении прямые, заданные линейными функциями y = tg ⁡ α 1 x + b 1 {\displaystyle y=\operatorname {tg} \alpha _{1}x+b_{1}} и y = tg ⁡ α 2 x + b 2 {\displaystyle y=\operatorname {tg} \alpha _{2}x+b_{2}} будут перпендикулярны, если выполнено условие α 2 = 1 2 π + α 1 {\displaystyle \alpha _{2}={\frac {1}{2}}\pi +\alpha _{1}} . Эти же прямые будут перпендикулярны, если tg ⁡ α 1 tg ⁡ α 2 = − 1 {\displaystyle \operatorname {tg} \alpha _{1}\operatorname {tg} \alpha _{2}=-1} . (Здесь α 1 , α 2 {\displaystyle \alpha _{1},\alpha _{2}} — углы наклона прямой к горизонтали)

Построение перпендикуляра

Шаг 1: (красный ) С помощью циркуля проведём полуокружность с центром в точке P, получив точки А» и В».

Шаг 2: (зелёный ) Не меняя радиуса, построим две полуокружности с центром в точках A» и В» соответственно, проходящими через точку Р. Кроме точки Р есть ещё одна точка пересечения этих полуокружностей, назовём её Q.{2}}}} ; y o = (x b − x a) ⋅ (x p − x o) (y b − y a) + y p {\displaystyle y_{o}={\frac {(x_{b}-x_{a})\cdot (x_{p}-x_{o})}{(y_{b}-y_{a})}}+y_{p}} .

В трёхмерном пространстве

Перпендикулярные прямые

Две прямые в пространстве перпендикулярны друг другу, если они соответственно параллельны некоторым двум другим взаимно перпендикулярным прямым, лежащим в одной плоскости. Две прямые, лежащие в одной плоскости, называются перпендикулярными (или взаимно перпендикулярными), если они образуют четыре прямых угла.

Перпендикулярность прямой к плоскости

Определение : Прямая называется перпендикулярной к плоскости, если она перпендикулярна всем прямым, лежащим в этой плоскости.

Признак : Если прямая перпендикулярна каждой из двух пересекающихся прямых плоскости, то она перпендикулярна этой плоскости.

Плоскость , перпендикулярная одной из двух параллельных прямых, перпендикулярна и другой. Через любую точку пространства проходит прямая, перпендикулярная к данной плоскости, и притом только одна.

Перпендикулярные плоскости

Две плоскости называются перпендикулярными, если двугранный угол между ними равен 90°.

В многомерных пространствах

Перпендикулярность плоскостей в 4-мерном пространстве

Перпендикулярность плоскостей в четырёхмерном пространстве имеет два смысла: плоскости могут быть перпендикулярны в 3-мерном смысле, если они пересекаются по прямой (а следовательно, лежат в одной гиперплоскости), и двугранный угол между ними равен 90°.

Плоскости могут быть также перпендикулярны в 4-мерном смысле, если они пересекаются в точке (а следовательно, не лежат в одной гиперплоскости), и любые 2 прямые, проведённые в этих плоскостях через точку их пересечения (каждая прямая в своей плоскости), перпендикулярны.

В 4-мерном пространстве через данную точку можно провести ровно 2 взаимно перпендикулярные плоскости в 4-мерном смысле (поэтому 4-мерное евклидово пространство можно представить как декартово произведение двух плоскостей).{k}} , то есть (∀ a → ∈ L 1) (∀ b → ∈ L k) a → b → = 0 {\displaystyle (\forall {\vec {a}}\in L_{1})\ (\forall {\vec {b}}\in L_{k})\ {\vec {a}}{\vec {b}}=0}

«Перпендикулярные прямые в пространстве.

Перпендикулярность прямой и плоскости»

Вариант 1

Уровень А

1. Какое утверждение верно?

1) Если одна из двух прямых перпендикулярна к третьей прямой, то и другая прямая перпендикулярна к этой прямой.

2) Если две прямые перпендикулярны к третьей прямой, то они параллельны.

3) Если две прямые перпендикулярны к плоскости, то они параллельны.

2. ABCD – прямоугольник, BM ┴ (ABC ) . Тогда неверно, что…

1) BM AC ;

2) AM AD ;

3) MD DC .

3. Прямая m перпендикулярна к прямым a и b , лежащим в плоскости α, но m не перпендикулярна к плоскости α. Тогда прямые a и b

1) параллельны;

2) пересекаются;

3) скрещиваются.

4. Плоскость α проходит через вершину А ромба ABCD перпендикулярно диагонали АС. Тогда диагональ BD …

1) перпендикулярна плоскости α;

2) параллельна плоскости α;

3) лежит в плоскости α.

5. a α , b ┴ α. Тогда прямые a и b не могут быть …

1) скрещивающимися;

2) перпендикулярными;

3) параллельными.

6. ABCD – параллелограмм, BD α, AC ┴ α. Тогда ABCD не может быть …

1) прямоугольником;

2) квадратом;

3) ромбом.

1) радиусам; 2) диаметрам; 3) хордам.

8. Какое утверждение верно:

1) Прямая и не проходящая через неё плоскость, перпендикулярные другой плоскости, параллельны между собой.

2) Плоскость и перпендикулярная данной плоскости, перпендикулярна и к прямой, параллельной данной плоскости.

3) Плоскость, перпендикулярная данной прямой, перпендикулярна и к плоскости, параллельной данной прямой.

9. AC ┴ (BDM ) . Тогда отрезок BM в треугольнике АВС является …

1) медианой;

2) высотой;

3) биссектрисой.

Вариант 1

https://pandia.ru/text/78/082/images/image006_123.gif»>(а, ВМ ) = …

https://pandia.ru/text/78/082/images/image003_184.gif»> α , СМ = МВ, АМ = 2,5 см, АС = 3 см. Тогда АВ = …

https://pandia.ru/text/78/082/images/image009_91.gif»>см. АС BD = О . FO ┴ (ABC ), FO = см. Расстояние от точки F до вершины квадрата равно …

https://pandia.ru/text/78/082/images/image013_21.jpg» align=»left»>

5. ABCD – прямоугольник. BF ┴ (ABC ). CF = 20 см, DF = 25 см. Тогда длина отрезка CD равна …

https://pandia.ru/text/78/082/images/image015_17.jpg» align=»left»>лежит в плоскости α .

5. ABCD — параллелограмм, АВhttps://pandia.ru/text/78/082/images/image016_17.jpg» align=»left»>скрещивающимися.

7. Dhttps://pandia.ru/text/78/082/images/image006_123.gif»> (АВ, CD) =600.

8. Какое утверждение неверное?

1) Через любую точку пространства проходит прямая, перпендикулярная к данной плоскости, и притом только одна.

2) Через точку, не лежащую на данной прямой, можно построить только одну плоскость, перпендикулярную данной прямой.

3) Через точку, не лежащую на данной прямой, можно построить только одну прямую, перпендикулярную данной прямой.

13.11.2016 14:35

Тестовые задания по геометрии к разделу «Прямые и плоскости в пространстве»1.Аксиомы стереометрии. 2.Параллельность прямых и плоскостей. 3.Перпендикулярность прямых и плоскостей. Ответы в конце разработки

Просмотр содержимого документа


«Тестовые задания по геометрии к разделу «Прямые и плоскости в пространстве» 1 курс СПО»

Раздел № 3.

Прямые и плоскости в пространстве

Предмет стереометрии. Основные понятия и аксиомы стереометрии.

Пространственные фигуры.

Параллельность прямыхв пространстве. Параллельность двух плоскостей.

Векторы в пространстве.

Параллельный перенос.

Сечение многогранников.

Перпендикулярность прямых, прямой и плоскости.

Перпендикуляр и наклонная.

Угол между прямой и плоскостью.

Двугранный угол. Перпендикулярность плоскостей.

Аксиомы стереометрии

Вариант 1

1) АВС 2) DBC 3) DAB 4) DAC

Каким плоскостям принадлежит точка К?

1) АВС и ABD

Выберите верные высказывания:

1) Любые три точки лежат в одной плоскости.

2) Если центр окружности и ее точка лежат в плоскости, то и вся окружность лежит в этой плоскости.

3) Через три точки, лежащих на прямой, проходит только одна плоскость.

4) Через две пересекающихся прямые проходит плоскость, и притом только одна.

Ответ: ______

Выберите неверные высказывания:

1) Если три прямые имеют общую точку, то они лежат в одной плоскости.

3) Две плоскости могут имеет только две общие точки.

4) Три попарно пересекающиеся в разных точках прямые, лежат в одной плоскости.

Ответ: ______

Назовите прямую, по которой пересекаются плоскости A 1 BC и A 1 AD.

1) DC 2) A 1 D 1

3) D 1 D 4) D 1 C

Назовите прямую, по которой пересекаются плоскости DCC 1 и A 1 AD.

1) DC 2) A 1 D 1

3) D 1 D 4) D 1 C

Прямые АВ и CD пересекаются. Через прямую АВ проведена плоскость. Назовите линию пересечения данной плоскости с плоскостью ВСD.

1) АС 2) АB 3) BС 4) ВD

Прямые АВ и CD пересекаются. Через точки В и D проведена плоскость. Назовите линию пересечения данной плоскости с плоскостью AСD.

1) АС 2) АB 3) BС 4) ВD

Вариант 2

Точка Р лежит на прямой МN. Назовите плоскость, которой принадлежит точка Р.

1) АВС 2) DBC 3) DAB 4) DAC

Каким плоскостям принадлежит точка F?

1) АВС и ACD

Выберите верные высказывания:

1) Любые четыре точки лежат в одной плоскости.

2) Через прямую и не лежащую на ней точку проходит только одна плоскость.

3) Если три точки окружности лежат в плоскости, то и вся окружность лежит в этой плоскости.

4) Две плоскости могут иметь только одну общую точку.

Ответ: ______

Выберите неверные высказывания:

1) Две окружности, имеющие общий центр, лежат в одной плоскости.

3) Три вершины треугольника принадлежат одной плоскости.

4) Через две параллельные прямые проходит плоскость, и притом только одна.

Ответ: ______

Назовите прямую, по которой пересекаются плоскости DCC 1 и A 1 BC.

1) DC 2) A 1 D 1

3) D 1 D 4) D 1 C

Назовите прямую, по которой пересекаются плоскости ABC и C 1 CB.

1) BC 2) B 1 C 1

3) A 1 B 4) B 1 B

Прямые АВ и CD пересекаются. Через прямую CD проведена плоскость. Назовите линию пересечения данной плоскости с плоскостью AВС.

1) СD 2) АD 3) BС 4) ВD

Прямые АВ и CD пересекаются. Через точки A и D проведена плоскость. Назовите линию пересечения данной плоскости с плоскостью BСD.

1) АС 2) АD 3) BС 4) ВD

Вариант 1

Точки М, Р, К – середины ребер DA, DB, DC тетраэдра DABC. Назовите прямую, параллельную плоскости FBC.

1) МР 2) РК 3) МК 4) МК и РК

АВСDA 1 B 1 C 1 D 1 – прямоугольный параллелепипед. Какая из прямых параллельна плоскости A 1 B 1 C 1 ?

1) а 2) b 3) p 4) m

В тетраэдре DАВС ВК = КС, DP = PC. Плоскости какой грани параллельна прямая РК?

1) DAB 2) DBC 3) DAC 4) ABC

Выберите верные высказывания:

1) Две прямые в пространстве называются параллельными, если они не пересекаются.

2) Если одна из двух параллельных прямых параллельна плоскости, то другая прямая либо так же ей параллельна, либо лежит в этой плоскости.

3) Существует такая прямая, которая лежит в плоскости и параллельна прямой, пересекающей данную плоскость.

4) Скрещивающиеся прямые не имеют общих точек.

Ответ: ______


1) a || n 2) a || b

3) b || c 4) a || c

верные высказывания:

1) Прямые СD и MN скрещивающиеся.

2) Прямые АВ и MN лежат в одной плоскости.

3) Прямые СD и MN пересекаются.

4) Прямые АВ и СD скрещивающиеся.

Ответ: ______


1) a и b пересекающиеся прямые

2) a и b параллельные прямые

3) a и b скрещивающиеся прямые

Определите взаимное расположение прямых.

1) a и b пересекающиеся прямые

2) a и b параллельные прямые

3) a и b скрещивающиеся прямые

Треугольники АВК и АВF расположены так, что прямые АВ и FK скрещиваются. Как расположены прямые АК и ВF?

В тетраэдре DАВС АВ = ВС = АС = 20; DA = DB = DC = 40. Через середину ребра АС плоскость, параллельная АD и ВC. Найдите периметр сечения.

Ответ: ____

Параллельность прямых и плоскостей

Вариант 2

Точки М, Р, К – середины ребер DA, DB, DC тетраэдра DABC. Назовите прямую, параллельную плоскости FАB.

1) МР 2) РК 3) МК 4) МК и РК


АВСDA 1 B 1 C 1 D 1 – прямоугольный параллелепипед. Какая из прямых параллельна плоскости A 1 AD?

1) а 2) b 3) p 4) m

В тетраэдре DАВС AM = MD, AN = NB. Плоскости какой грани параллельна прямая MN?

1) DAB 2) DBC 3) DAC 4) ABC

Выберите верные высказывания:

1) Параллельные прямые не имеют общих точек.

2) Если прямая параллельна данной плоскости, то она параллельна любой прямой, лежащей в этой плоскости.

3) Если прямая параллельна линии пересечения двух плоскостей и не принадлежит ни одной из них, то она параллельна каждой из этих плоскостей.

4) Существует параллелепипед, у которого все углы граней острые.

Ответ: ______

Точки А, В, С и D – середины ребер прямоугольного

параллелепипеда. Назовите параллельные прямые.


1) a || n 2) a || b

3) b || c 4) a || c

Точки А и D – середины ребер параллелепипеда. Выберите верные высказывания:

1) Прямые СD и MN пересекаются.

2) Прямые АВ и MN скрещивающиеся

3) Прямые АВ и СD параллельные.

4) Прямые АВ и MN пересекаются

Ответ: ______


Определите взаимное расположение прямых.

1) a и b пересекающиеся прямые

2) a и b параллельные прямые

3) a и b скрещивающиеся прямые

Точки А и В – середины ребер параллелепипеда. Определите взаимное расположение прямых.

1) a и b пересекающиеся прямые

2) a и b параллельные прямые

3) a и b скрещивающиеся прямые

Два равнобедренных треугольника АВС и АВD с общим основанием АВ расположены так, что точка С не лежит в плоскости АВD. Определите взаимное расположение прямых, содержащих медианы треугольников, проведенных к сторонам ВС и ВD.

1) они параллельны 2) скрещиваются 3) пересекаются

В тетраэдре DАВС АВ = ВС = АС = 10; DA = DB = DC = 20. Через середину ребра ВС плоскость, параллельная АС и ВD. Найдите периметр сечения.

Ответ: ____

Вариант 1

Через сторону АВ треугольника АВС проведена плоскость, перпендикулярная к стороне ВС. Определите вид треугольника относительно углов.

Треугольник АВС – правильный, О – центр треугольника. Расстояние от точки М до вершины А равно 3. Найдите высоту треугольника.

Ответ: ____

АВСD – параллелограмм; Найдите периметр параллелограмма.

1) 20 2) 25 3) 40 4) 60

Через вершину А треугольника ABC проведена плос­кость α, параллельная ВС. Расстояние от ВС до плоскости α равно 12. Найдите расстояние от точки пересечения ме­диан треугольника АВС до этой плоскости.

1) 8 2) 6 3) 12 4) 18

Высота ромба равна 12. Точка М равноудалена от всех сторон ромба и находится на расстоянии, равном 8, от его плоскости. Чему равно расстояние точки М до сторон ромба?

Ответ: ____

Выберите верные высказывания:

2) Две прямые, перпендикулярные к одной плоскости, параллельны.

3) Длина перпендикуляра меньше длины наклонной, проведенной из той же точки.

4) Две скрещивающиеся прямые могут быть перпендикулярными к одной плоскости.

Ответ: ______

Отрезок АВ упирается концами А и В в грани прямого двугранного угла. Расстояния от точек А и В до ребра равны 1, а длина отрезка АВ равна 3. Найдите длину про­екции этого отрезка на ребро.

В тетраэдре DABC АО пресекает ВС в точке Е; Найдите.

Прямоугольник ABCD и параллелограмм ВЕМС распо­ложены так, что их плоскости взаимно перпендикулярны. Найдите угол MCD.

Перпендикулярность прямых и плоскостей

Вариант 2

Через сторону АD параллелограмма АВСD, проведена плоскость, перпендикулярная к стороне DС. Определите вид треугольника АВС.

1) остроугольный 2) прямоугольный 3) тупоугольный

Треугольник АВС – правильный, О – центр треугольника. Высота треугольника равна 3. Найдите расстояние от точки М до вершин треугольника.

Ответ: ____

АВСD – параллелограмм; Найдите BD.

1) 20 2) 15 3) 40 4) 10

Через вершину А треугольника ABC проведена плос­кость α, параллельная ВС. Расстояние от точки пересече­ния медиан треугольника АВС до этой плоскости равно 4. На каком расстоянии от плоскости находится ВС?

1) 8 2) 6 3) 12 4) 14

Точка Р удалена от всех сторон ромба на расстояние» равное, и находится от его плоскости на расстоянии равном 2. Чему равна сторона ромба, если его угол 30°?

Ответ: ____

На рисунке Найдите угол между МС и плоскостью АМВ.

1) 30 0 2) 60 0 3) 90 0 4) 45 0

Выберите верные высказывания:

1) Угол между прямой и плоскостью может быть не больше 90 0 .

2) Две плоскости, перпендикулярные к одной прямой, пересекаются.

3) Длина перпендикуляра больше длины наклонной, проведенной из той же точки.

4) Диагональ прямоугольного параллелепипеда больше любого из ребер.

Ответ: ______

Отрезок АВ упирается концами А и В в грани прямого двугранного угла. Расстояния от точек А и В до ребра равны 2, а длина отрезка АВ равна 4. Найдите длину про­екции этого отрезка на ребро.

В тетраэдре DABC основание ABC — правильный тре­угольник. Вершина D проецируется в его центр О. Найди­те угол между плоскостью ADO и гранью DCB.

1) 30 0 2) 60 0 3) 90 0 4) 45 0

Треугольник АМВ и прямоугольник ABCD расположе­ны так, что их плоскости взаимно перпендикулярны. Най­дите угол MAD.

1) 90 0 2) 60 0 3) 30 0 4) 45 0

Тест 1

Вариант 1

Вариант 2

Тест 2

Вариант 1

Вариант 2

Тест 3

Вариант 1

Вариант 2

Перпендикулярность в пространстве могут иметь:

1. Две прямые

3. Две плоскости

Давай по очереди рассмотрим эти три случая: все относящиеся к ним определения и формулировки теорем. А потом обсудим очень важную теорему о трёх перпендикулярах.

Перпендикулярность двух прямых.

Определение:

Ты можешь сказать: тоже мне, открыли Америку! Но вспомни, что в пространстве всё не совсем так, как на плоскости.

На плоскости перпендикулярными могут оказаться только такие прямые (пересекающиеся):

А вот перпендикулярность в пространстве двух прямых может быть даже в случае если они не пересекаются. Смотри:

прямая перпендикулярна прямой, хотя и не пересекается с нею. Как так? Вспоминаем определение угла между прямыми: чтобы найти угол между скрещивающимися прямыми и, нужно через произвольную точку на прямой a провести прямую. И тогда угол между и (по определению!) будет равен углу между и.

Вспомнили? Ну вот, а в нашем случае — если окажутся перпендикулярны прямые и, то нужно считать перпендикулярными прямые и.

Для полной ясности давай рассмотрим пример. Пусть есть куб. И тебя просят найти угол между прямыми и. Эти прямые не пересекаются — они скрещиваются. Чтобы найти угол между и, проведём.

Из-за того, что — параллелограмм (и даже прямоугольник!), получается, что. А из-за того, что — квадрат, выходит, что. Ну, и значит.

Перпендикулярность прямой и плоскости.

Определение:

Вот картинка:

прямая перпендикулярна плоскости, если она перпендикулярна всем-всем прямым в этой плоскости: и, и, и, и даже! И ещё миллиарду других прямых!

Да, но как же тогда вообще можно проверить перпендикулярность в прямой и плоскости? Так и жизни не хватит! Но на наше счастье математики избавили нас от кошмара бесконечности, придумав признак перпендикулярности прямой и плоскости .

Формулируем:

Оцени, как здорово:

если найдутся всего лишь две прямые (и) в плоскости, которым перпендикулярна прямая, то эта прямая сразу окажется перпендикулярна плоскости, то есть всем прямым в этой плоскости (в том числе и какой-то стоящей сбоку прямой). Это очень важная теорема, поэтому нарисуем её смысл ещё и в виде схемы.

И опять рассмотрим пример .

Пусть нам дан правильный тетраэдр.

Задача: доказать, что. Ты скажешь: это же две прямые! При чём же здесь перпендикулярность прямой и плоскости?!

А вот смотри:

давай отметим середину ребра и проведём и. Это медианы в и. Треугольники — правильные и.

Вот оно, чудо: получается, что, так как и. И далее, всем прямым в плоскости, а значит, и. Доказали. И самым главным моментом оказалось именно применение признака перпендикулярности прямой и плоскости.

Когда плоскости перпендикулярны

Определение:

То есть (подробнее смотри в теме «двугранный угол») две плоскости (и) перпендикулярны, если окажется, что угол между двумя перпендикулярами (и) к линии пересечения этих плоскостей равен. И есть теорема, которая связывает понятие перпендикулярных плоскостей с понятием перпендикулярность в пространстве прямой и плоскости.

Теорема эта называется

Критерий перпендикулярности плоскостей.

Давай сформулируем:

Как всегда, расшифровка слов «тогда и только тогда» выглядит так:

  • Если, то проходит через перпендикуляр к.
  • Если проходит через перпендикуляр к, то.

(естественно, здесь и — плоскости).

Эта теорема — одна из самых важных в стереометрии, но, к сожалению, и одна из самых непростых в применении.

Так что нужно быть очень внимательным!

Итак, формулировка:

И снова расшифровка слов «тогда и только тогда». Теорема утверждает сразу две вещи (смотри на картинку):

давай попробуем применить эту теорему для решения задачи.

Задача : дана правильная шестиугольная пирамида. Найти угол между прямыми и.

Решение:

Из-за того, что в правильной пирамиде вершина при проекции попадает в центр основания, оказывается, что прямая — проекция прямой.

Но мы знаем, что в правильном шестиугольнике. Применяем теорему о трёх перпендикулярах:

И пишем ответ: .

ПЕРПЕНДИКУЛЯРНОСТЬ ПРЯМЫХ В ПРОСТРАНСТВЕ. КОРОТКО О ГЛАВНОМ

Перпендикулярность двух прямых.

Две прямые в пространстве перпендикулярны, если угол между ними.

Перпендикулярность прямой и плоскости.

Прямая перпендикулярна плоскости, если она перпендикулярна всем прямым в этой плоскости.

Перпендикулярность плоскостей.

Плоскости перпендикулярны, если двугранный угол между ними равен.

Критерий перпендикулярности плоскостей.

Две плоскости перпендикулярны тогда и только тогда, когда одна из них проходит через перпендикуляр к другой плоскости.

Теорема о трех перпендикулярах:

Ну вот, тема закончена. Если ты читаешь эти строки, значит ты очень крут.

Потому что только 5% людей способны освоить что-то самостоятельно. И если ты дочитал до конца, значит ты попал в эти 5%!

Теперь самое главное.

Ты разобрался с теорией по этой теме. И, повторюсь, это… это просто супер! Ты уже лучше, чем абсолютное большинство твоих сверстников.

Проблема в том, что этого может не хватить…

Для чего?

Для успешной сдачи ЕГЭ, для поступления в институт на бюджет и, САМОЕ ГЛАВНОЕ, для жизни.

Я не буду тебя ни в чем убеждать, просто скажу одну вещь…

Люди, получившие хорошее образование, зарабатывают намного больше, чем те, кто его не получил. Это статистика.

Но и это — не главное.

Главное то, что они БОЛЕЕ СЧАСТЛИВЫ (есть такие исследования). Возможно потому, что перед ними открывается гораздо больше возможностей и жизнь становится ярче? Не знаю…

Но, думай сам…

Что нужно, чтобы быть наверняка лучше других на ЕГЭ и быть в конечном итоге… более счастливым?

НАБИТЬ РУКУ, РЕШАЯ ЗАДАЧИ ПО ЭТОЙ ТЕМЕ.

На экзамене у тебя не будут спрашивать теорию.

Тебе нужно будет решать задачи на время .

И, если ты не решал их (МНОГО!), ты обязательно где-нибудь глупо ошибешься или просто не успеешь.

Это как в спорте — нужно много раз повторить, чтобы выиграть наверняка.

Найди где хочешь сборник, обязательно с решениями, подробным разбором и решай, решай, решай!

Можно воспользоваться нашими задачами (не обязательно) и мы их, конечно, рекомендуем.

Для того, чтобы набить руку с помощью наших задач нужно помочь продлить жизнь учебнику YouClever, который ты сейчас читаешь.

Как? Есть два варианта:

  1. Открой доступ ко всем скрытым задачам в этой статье —
  2. Открой доступ ко всем скрытым задачам во всех 99-ти статьях учебника — Купить учебник — 899 руб

Да, у нас в учебнике 99 таких статей и доступ для всех задач и всех скрытых текстов в них можно открыть сразу.

Доступ ко всем скрытым задачам предоставляется на ВСЕ время существования сайта.

И в заключение…

Если наши задачи тебе не нравятся, найди другие. Только не останавливайся на теории.

“Понял” и “Умею решать” — это совершенно разные навыки. Тебе нужны оба.

Найди задачи и решай!

Название: Геометрия. 10-11 класс. Тесты

Пособие содержит тесты по основным темам курса геометрии 10-11 классов в двух вариантах — 8 тестов для 10 класса и 9 тестов для 11 класса.
Предлагаемые тесты учитель может использовать для контроля знаний учащихся перед проведением контрольной работы или в качестве контрольной. Учащиеся могут использовать тесты при самоподготовке к выпускным экзаменам, а также к вступительным экзаменам в ВУЗы.

В данной книге представлены проверочные тесты по геометрии для 10-11 классов. Она является продолжением аналогичной книги по геометрии для 7-9 классов. Тесты даются в двух вариантах — 8 тестов для 10 класса и 9 тестов для 11 класса.
Тесты целесообразно проводить один раз в месяц в качестве проверочных работ перед контрольными или заменяя их. Учитывая сложность отдельных заданий, на проведение полного теста должно отводиться два урока. Однако учитель может разбить тест на 2 части (по 4 задания в каждой) и провести его на двух разных уроках в разные дни. В этом случае учитель должен учитывать то обстоятельство, что задания расположены не по степени возрастания сложности (т. е., например, задание 3 может быть сложнее, чем задание 5), сделано это умышленно, чтобы учащиеся решали не только легкие задачи, но и пытались решать более сложные. Но учитель, просмотрев задания отдельного теста, может сам варьировать число и сложность заданий.
Учитывая своеобразие проведения проверочных тестов, когда приводимые ответы в какой-то степени облегчают решение задачи, учитель может на последующем уроке провести анализ работ, расставляя акценты на теоретических обоснованиях решения задач, проводя необходимые доказательства с целью выявления логической обоснованности выбора учеником ответа.
Последовательность материала дана в соответствии с учебником по геометрии для 7-11 классов А. В. Погорелова. Однако учителя, работающие по другим учебным пособиям, сделав необходимые корректировки, также могут использовать их в своей работе.

Содержание
Предисловие
10 класс
Тест 1. Аксиомы стереометрии. Следствия из аксиом
Тест 2. Параллельность в пространстве
Тест 3. Перпендикулярность в пространстве
Тест 4. Параллельность и перпендикулярность в пространстве
Тест 5. Координаты в пространстве
Тест 6. Углы между прямыми и плоскостями
Тест 7. Векторы
Тест 8. Итоговый
11 класс
Тест 1. Двугранные и линейные углы. Многогранные углы
Тест 2. Параллелепипед и призма
Тест 3. Пирамида. Усеченная пирамида
Тест 4. Цилиндр. Конус. Шар
Тест 5. Объемы многогранников
Тест 6. Объемы тел вращения
Тест 7. Комбинации фигур
Тест 8. Итоговый — 1
Тест 9. Итоговый — 2
Ответы

Бесплатно скачать электронную книгу в удобном формате, смотреть и читать:
Скачать книгу Геометрия. 10-11 класс. Тесты. Алтынов П.И. 2001 — fileskachat.com, быстрое и бесплатное скачивание.

Скачать pdf
Ниже можно купить эту книгу по лучшей цене со скидкой с доставкой по всей России.

Quia — Геометрия Глава 12 Словарь

900 10 смежных дуг
A B
набор всех точек вне круга снаружи круга
линия, которая находится в той же плоскости, что и круг, и пересекает окружность ровно в одной точке касательная к окружности
отрезок, концы которого лежат на окружности хорда
точка пересечения окружности или сферы с касательной или плоскостью точка касательная
линия, пересекающая окружность в двух точках секущая
линия, касательная к двум окружностям общая касательная
совокупность всех точек внутри окружности внутренняя часть окружность
отрезок секущей, лежащий снаружи круга с одной конечной точкой на окружности внешняя секущая se gment
угол, вершина которого находится на окружности и стороны которого содержат хорды окружности вписанный угол
, где концы сегмента или дуги лежат на сторонах угла subtend
угол, вершиной которого является центр окружности центральный угол
дуга окружности, точки которой находятся на или снаружи от центрального угла большая дуга
непрерывная часть окружности состоящий из двух точек на окружности, называемых конечными точками, и всех точек окружности между ними дуги
дуги окружности, точки которой находятся внутри или внутри центрального угла вспомогательной дуги
расстояние по дуге, измеренное в линейных единицах длина дуги
две дуги одной окружности, пересекающиеся точно в одной точке
две дуги, которые находятся в одной или конгруэнтной окружности и имеют одинаковую длину конгруэнтные дуги
область внутри окружности, ограниченная хордой и дугой сегмент окружности
две окружности с совпадающими радиусами конгруэнтные окружности
две копланарные окружности, пересекающиеся ровно в одной точке касательные окружности
компланарные окружности с одинаковым центром концентрические окружности
дуга окружность, концы которой лежат на диаметре полукруг
область внутри окружности, ограниченная двумя радиусами окружности и их пересеченной дугой сектор окружности

геометрия — касательная плоскость к трем окружностям

Хорошо, чтобы продолжить мой последний пост, вот мой прогресс.

Предположим, что $ \ vec T_i $ — центры окружностей, векторы $ \ vec n_i $ нормальные к каждой окружности, а $ r_i $ — их соответствующие радиусы.

Первый шаг — спроецировать одну окружность на плоскость другой (в моем случае окружность 1 на плоскость окружности 3). Для краткости я буду использовать следующую функцию $ project (\ vec v_1, \ vec v_2) = \ vec v_1 — (\ vec v_2 \ cdot \ vec v_1) \ cdot \ vec v_2 $.

$ \ vec T’_1 = project (\ vec T_1, \ vec n_3) $ — точка проекции центра окружности на плоскость

$ \ vec T’_1 = \ vec T_1 + \ left [\ vec n_3 \ cdot \ left (\ vec T_3 — \ vec T_1 \ right) \ right] \ cdot \ vec n_3 $ — точка проекции центра круг на плоскости

$ \ vec y_1 = \ frac {project (\ vec n_1, \ vec n_3)} {\ vert project (\ vec n_1, \ vec n_3) \ vert} $ — ось y новой системы координат

$ \ vec x_1 = \ frac {\ vec n_3 \ times \ vec y_1} {\ vert \ vec n_3 \ times \ vec y_1 \ vert} $ — ось x новой системы координат

$ \ vec x_1 = \ frac {\ vec y_1 \ times \ vec n_3} {\ vert \ vec y_1 \ times \ vec n_3 \ vert} $ — ось x новой системы координат

$ f = \ vec n_1 \ cdot \ vec n_3 $ — отношение малой оси к радиусу окружности (большая ось)

Получение координат третьего круга в новой (2D) системе координат:

$ x_3 = \ left (\ vec T_3- \ vec T’_1 \ right) \ cdot \ vec x_1 $

$ y_3 = \ left (\ vec T_3- \ vec T’_1 \ right) \ cdot \ vec y_1 $

Теперь у нас есть две параллельные системы координат $ x, y $ и $ x_k, y_k $ со сдвигом $ (x_3, y_3) $.2 \ right] $

Решение этого уравнения даст четыре значения x на проецируемом эллипсе (координаты y должны быть вычислены, имея в виду, что, вообще говоря, $ x_3 \ cdot y_3> 0 \ Rightarrow x \ cdot y <0 $, и наоборот, в то время как $ x_k $ и $ y_k $ также должны быть вычислены, $ x \ cdot x_k> 0 $ для внешних и $ x \ cdot x_k <0 $ для внутренних касательных).

Этот метод до сих пор был проверен на моделях САПР. Осталось « спроецировать » точки эллипса на исходную плоскость (плоскость круга 1), и с соответствующими точками на окружности 3 у нас есть четыре линии, и каждая из этих линий будет лежать в двух из восьми. самолеты.Чтобы решить остальную часть проблемы, я думаю, что лучше всего было бы провести плоскость, перпендикулярную каждой линии, через центральную точку третьего круга (круг 2), спроецировать этот круг на эту плоскость и найти касательные к этому эллипсу из точки в какая линия пересекает плоскость. Получение уравнений плоскостей оттуда должно быть несложным … Если кому-то все еще интересно, я опубликую следующую процедуру, когда закончу и проверю ее.

РЕДАКТИРОВАТЬ Я исправил первое и третье уравнение, потому что, хотя они работали в 2D, это правильная форма и работает в 3D.2} $

$ \ vec {{TP_3} _i} = \ vec T_3 + {x_k} _i \ cdot \ vec x_1 + {y_k} _i \ cdot \ vec y_1 $ — точка касания на окружности 3

$ \ vec {{TP´_1} _i} = \ vec T´_1 + xT_i \ cdot \ vec x_1 + yT_i \ cdot \ vec y_1 $ — точка касания на спроецированном эллипсе

$ \ vec {{TP_1} _i} = \ vec {{TP´_1} _i} + \ frac {\ vec n_1 \ left (\ vec T_1 — \ vec {{TP´_1} _i} \ right)} { \ vec n_1 \ cdot \ vec n_3} \ vec n_3 $ — точка касания на окружности 1 (точка на эллипсе, «непроектированная» на плоскость окружности 1)

$ \ vec {{l_1} _i} = \ frac {\ vec {TP_3} _i — \ vec {TP_1} _i} {\ vert \ vec {TP_3} _i — \ vec {TP_1} _i \ vert} $ — касательная линия на кругах 1 и 3

Теперь мы установили плоскость (перпендикулярно линии $ \ vec {{l_1} _i} $, через точку $ T_2 $) и систему координат:

$ \ vec {{y_2} _i} = \ frac {project (\ vec n_2, \ vec {{l_1} _i})} {\ vert project (\ vec n_2, \ vec {{l_1} _i}) \ vert } $ $ \ quad, \ quad $ $ \ vec {{x_2} _i} = \ frac {\ vec {{y_2} _i} \ times \ vec n_2} {\ vert \ vec {{y_2} _i} \ times \ vec n_2 \ vert}

$

$ \ vec {{TP´_3} _i} = \ vec {{TP_3} _i} + \ left [\ vec {{l_1} _i} \ cdot \ left (\ vec T_2 — \ vec {{TP_3} _i} \ right) \ right] \ cdot \ vec {{l_1} _i} $ — проекция $ \ vec {{TP_3} _i} $ на плоскость

Координаты проецируемой точки в 2D системе координат:

$ {x_ {TP}} _ i = \ left (\ vec {{TP´_3} _i} — \ vec T_2 \ right) \ cdot \ vec {x_2} _i $

$ {y_ {TP}} _ i = \ left (\ vec {{TP´_3} _i} — \ vec T_2 \ right) \ cdot \ vec {y_2} _i $

$ f´_i = \ vert \ vec {{l_1} _i} \ cdot \ vec n_2 \ vert $ — отношение малой оси к радиусу окружности (большой оси)

Теперь мы можем вычислить координаты точек касания (лучей от $ {TP´_3} _i $) на эллипсе:

$ _1 {{xT2} _i} _2 = \ frac {f´_i ^ 2 r_2 ^ 2 {x_ {TP}} _ i — r_2 {y_ {TP}} _ i \ pm \ sqrt {f´_i ^ 2 \ left ({x_ {TP}} _ i ^ 2 — r_2 ^ 2 \ right) + {y_ {TP}} _ i ^ 2}} {f´_i ^ 2 {x_ {TP}} _ i ^ 2 + {y_ {TP} } _i ^ 2}

доллар

Для каждого из этих $ xT2_i $ мы получаем соответствующий $ yT2_i = \ pm f´_i \ sqrt {r_2 ^ 2 — xT2_i ^ 2} $.И каждая из этих точек будет «непроектирована» на плоскость круга.

$ \ vec {TP´_2} _i = \ vec T_2 + xT2_i \ cdot \ vec {x_2} _i + yT2_i \ cdot \ vec {y_2} _i $

$ \ vec {{TP_2} _i} = \ vec {TP´_2} _i + \ left [\ vec n_2 \ cdot \ left (\ vec T_2 — \ vec {TP´_2} _i \ right) \ right] \ cdot \ vec n_2 $

$ \ vec {l_2} _i = \ frac {\ vec {{TP_3} _i} — \ vec {{TP_2} _i}} {\ vert \ vec {{TP_3} _i} — \ vec {{TP_2} _i} \ vert} $ — вторая прямая в касательной плоскости (sic. , есть две такие прямые для каждого i … получается восемь самолетов)

$ \ vec {n_1} _i = \ frac {\ vec {l_1} _i \ times \ vec {l_2} _i} {\ vert \ vec {l_1} _i \ times \ vec {l_2} _i \ vert} $ — нормально вектор к плоскости

$ {D_1} _i = — \ vec {n_1} _i \ cdot \ vec {{TP_3} _i} $ — постоянная плоскости

$ \ vec {n_1} _i \ cdot \ vec T + {D_1} _i = 0 $ — уравнение касательных плоскостей

Относительное положение двух окружностей

Этот урок проиллюстрирует довольно простую концепцию — взаимное расположение двух окружностей.То есть, чтобы определить, лежат ли две окружности с известными центрами и радиусами полностью вне друг друга, касаются ли друг друга, пересекаются в двух точках или таковы, что одна находится внутри другой.

Для всех этих случаев рассмотрим две окружности с центрами C 1 и C 2 и радиусами r 1 и r 2 соответственно.

Две окружности, касаясь друг друга снаружи

Я немного говорил об этом случае на предыдущем уроке. Взгляните на рисунок ниже.

У нас есть два круга, внешне соприкасающиеся друг с другом. Совершенно очевидно, что расстояние между центрами двух окружностей равно сумме их радиусов. То есть C 1 C 2 = r 1 + r 2 , что будет необходимым условием для того, чтобы два круга касались друг друга снаружи. Имеет смысл?

Две окружности, лежащие друг напротив друга

Что если в предыдущем случае мы немного отодвинем один из кругов от другого? Два круга больше не будут касаться друг друга и лежат вне друг друга.

Поскольку, разделив их, мы увеличили расстояние между их центрами, расстояние C 1 C 2 станет больше, чем r 1 + r 2 . Следовательно, необходимое условие будет: C 1 C 2 > r 1 + r 2

Два круга, внутренне соприкасающихся друг с другом

Посмотрите на рисунок.

В этом случае расстояние между центрами равно разнице радиусов окружностей, т.е.е. C 1 C 2 = r 1 — r 2 . Вы видите это?

Один круг лежит внутри другого

Теперь сдвинем меньший круг немного вправо, чтобы он полностью лежал внутри большего круга.

При перемещении круга мы делаем C 1 C 2 меньше, чем в предыдущем случае. Это означает, что необходимое условие для этого случая будет: C 1 C 2 1 — r 2 .

Две окружности, пересекающиеся в двух точках

Рассмотрим еще раз третий случай. Что, если мы сдвинем меньший кружок влево? (Или больший справа?). Теперь круги будут пересекаться в двух точках. И теперь мы увеличиваем расстояние между их центрами, подразумевая C 1 C 2 > r 1 — r 2 .

Но мы не можем толкать его бесконечно влево, потому что возникнет ситуация, когда C 1 C 2 станет равным r 1 + r 2 , и круги снова коснутся друг друга, но внешне .Это означает, что если мы хотим, чтобы круги пересекались в двух точках, мы должны сохранить расстояние между их центрами меньше суммы их радиусов.

Учитывая все это, получаем необходимое условие для этого случая: r 1 — r 2 1 C 2 1 + r 2

Обратите внимание, что в последних трех случаях я предположил, что r 1 > r 2 (в противном случае r 1 — r 2 будет отрицательным).Для любых двух заданных окружностей вы всегда можете предположить, что больший радиус равен r 1 , или, в качестве альтернативы, вы можете использовать абсолютное значение их разности.

И это все о взаимном расположении двух окружностей. Надеюсь, это было довольно легко понять.

Резюме урока

Я резюмировал урок в следующей таблице:

Позиция

Состояние

Лежат друг за другом
C
1 C 2 > r 1 + r 2
Касание снаружи
C
1 C 2 = r 1 + r 2
Пересечение в двух точках
| r
1 — r 2 | 1 C 2 1 + r 2
Касание внутри
C
1 C 2 = | r 1 — r 2 |
Один лежит внутри другого
C
1 C 2 <| r 1 — r 2 |

В следующем уроке я приведу несколько примеров, чтобы продемонстрировать эти случаи.

Искусство решения проблем

Введение

Теория радикальной оси — бесценный геометрический инструмент, с помощью которого можно довольно легко решать сложные геометрические задачи. Проблемы, связанные с этим, можно найти на многих крупных олимпиадах по математике, в том числе на престижных соревнованиях USAMO. Поэтому любому начинающему математическому олимпийцу следует внимательно изучить этот материал, так как он может содержать ключи к его будущему успеху.

Не все теоремы будут полностью доказаны в этом тексте.Цель этого документа — познакомить вас с некоторыми ключевыми концепциями, а затем дать вам возможность получить некоторые из прекрасных результатов самостоятельно. Таким образом, вы поймете и сохраните содержащуюся здесь информацию гораздо более надежно. Наконец, ваши новообретенные знания будут проверены на нескольких сложных задачах, которые являются наглядными примерами того, как можно использовать теорию радикальной оси и почему она имеет отношение к этой ситуации. Я надеюсь, что после того, как вы прочитаете этот текст, вы станете лучше изучающим математику, вооружившись еще одним инструментом для решения сложных задач.Но, в любом случае, удачи.

Определения

Степень точки относительно окружности (с радиусом и центром), которая впоследствии будет дублироваться, определяется как равная.

Обратите внимание, что степень точки отрицательна, если точка находится внутри круга.

Коренная ось двух неконцентрических окружностей определяется как геометрическое место точек, в которых степени по и равны. Другими словами, если — центр и радиус, то точка находится на радикальной оси тогда и только тогда, когда

Результаты

Теорема 1: (Степень точки) Если линия, проведенная через точку P, пересекает окружность в точках A и B, то.

Теорема 2: (Теорема о радикальной оси)

а. Коренная ось — это линия, перпендикулярная линии, соединяющей центры окружностей (линия).

г. Если два круга пересекаются в двух общих точках, их радикальная ось — это линия, проходящая через эти две точки.

г. Если они пересекаются в одной точке, их радикальная ось является общей внутренней касательной.

г. Если круги не пересекаются, и если одна не полностью содержит другую, их радикальная ось является перпендикуляром к проходящей через точку A, единственной точке на такой, что.

Теорема 3: (Теорема о радикальном совпадении осей) Три попарные радикальные оси трех окружностей совпадают в точке, называемой радикальным центром .

Пробки

Теорема 1 является тривиальной степенью точки, поэтому она предоставляется читателю в качестве упражнения. (Подсказка: проведите линию через P и центр.)

Теорема 2 будет доказана здесь. Предположим, что окружности и с центрами и и радиусами и соответственно. (Возможно, вам стоит нарисовать здесь несколько кругов.)

Сначала займемся деталью (б). Предположим, что круги пересекаются в точках и точка P лежит на. Тогда по теореме 1 степени P относительно обеих окружностей равны, а значит, транзитивны. Таким образом, если точка P лежит на, то степени P по обеим окружностям равны.

Теперь мы докажем обратное только что доказанному утверждению; поскольку обратное эквивалентно обратному, тогда тогда и только тогда будет доказано. Предположим, что P не лежит на. В частности, линия не пересекает X.Затем пересекает круги и второй раз в разных точках и соответственно. (Если, например, касается касательного, мы принимаем соглашение, которое; аналогичные соглашения справедливы для. Величина точки все еще сохраняется в этом случае. Также обратите внимание, что и не могут оба равны, так как не могут касаться обоих кругов.) не равно, не равно и, следовательно, по теореме 1 не совпадает с, как хотелось бы. Это завершает часть (b).

Для остальных частей воспользуемся леммой:

Лемма 1: Позвольте быть точкой на плоскости, и пусть будет основанием перпендикуляра от до.Затем .

Доказательство леммы представляет собой простое приложение теоремы Пифагора и снова будет оставлено читателю в качестве упражнения.

Лемма 2: На прямой существует единственная точка P такая, что.

Доказательство: сначала покажите, что P лежит между доказательством от противного и через него, используя немного теории неравенства и того факта, что. Затем используйте тот факт, что (константа), для доказательства леммы.

Лемма 1 показывает, что каждая точка на плоскости может быть эквивалентно отображена на прямую на.Лемма 2 показывает, что только одна точка в этом отображении удовлетворяет данному условию. Объединение этих двух лемм показывает, что радикальная ось является прямой, перпендикулярной к, завершающей часть (а).

Части (c) и (d) оставим читателю в качестве упражнения. (Также попробуйте доказать часть (b), используя только леммы.)

А теперь попробуйте доказать теорему 3 самостоятельно! (Подсказка: пусть P будет пересечением двух радикальных осей.)

Упражнения

Если вы еще этого не сделали, докажите теоремы и леммы, изложенные в разделе доказательств. Примечание: Следующие проблемы не решаются. Если застряли, спросите на форуме.

Задача 1. Две окружности P и Q пересекаются в точках X и Y. Точка A расположена так, что AP = 10 и AQ = 15 12. Если радиус Q равен 7, найдите радиус P. Примечание. Ошибка в этой проблеме ранее делала ее неразрешимой.

Проблема 2. Решите 2009 USAMO Проблема 1. Если вы уже знаете, как ее решить.

Задача 3. Две окружности P и Q с радиусами 1 и 2, соответственно, пересекаются в точках X и Y. Окружность P находится слева от круга Q. Докажите, что точка A находится слева от круга тогда и только тогда, когда.

Задача 4. Решите 2012 USAJMO Задачу 1.

Задача 5. Применима ли теорема 2 к окружностям, в которых одна содержится внутри другой? Как насчет внутренних касательных кругов? Концентрические круги?

Задача 6. Постройте радикальную ось двух окружностей. Что произойдет, если один круг охватит другой?

Задача 7. Решите проблему 1 IMO 1995 года двумя разными способами. Сравните свои решения с предложенными решениями.

Math Wizard — «ГЕОМЕТРИЧЕСКОЕ ОПРЕДЕЛЕНИЕ ТЕРМИНОВ (ОБЗОР)» …

«ГЕОМЕТРИЧЕСКОЕ ОПРЕДЕЛЕНИЕ ТЕРМИНОВ (РЕЗЮМЕ)»

ГЕОМЕТРИЯ –Гео означает «земля», а метрон означает «измерение».
Острие — не имеет длины, ширины или толщины.
Line — имеет бесконечную длину, но не имеет ширины и толщины.
Плоскость — бесконечная длина и ширина, но без толщины. Ровная поверхность.
Пространство — это набор всех точек
Рисунок — это любой набор точек
Копланарность — Набор точек, лежащих в одной плоскости.
Некопланарные точки — точки, не лежащие в одной плоскости.
Коллинеарные точки — это точки, лежащие на одной прямой.
Неколлинеарность — точки, не лежащие на одной линии.
Пересекающиеся линии — две линии, имеющие общую точку.
Перпендикуляр — две прямые пересекаются и образуют прямые углы.
Параллельные прямые — копланарные прямые, которые не пересекаются.
Параллельные прямые — 3 или более прямых, имеющих общую точку.
Параллельные плоскости — плоскости, не имеющие общей точки.
Косые линии — две линии, не лежащие в одной плоскости.
Пересекающиеся плоскости — это плоскости, которые пересекаются по линии.
Сегмент — имеет определенные конечные точки.
Луч — линия с начальной точкой, но без конечной точки
Конечная точка —
Средняя точка (на полпути) — M — это средняя точка линии AB.
Биссектриса сегмента — это сегмент, луч, линия или плоскость, которые пересекают сегмент в его средней точке.
Угол — образован двумя неколлинеарными лучами, имеющими общую конечную точку.
Конечная точка — это вершина угла, а каждый луч — это сторона угла.
Транспортир — используется для измерения угла.
Острый угол — размер менее 90 градусов.
Прямой угол — размер равен 90 градусам. Стороны перпендикулярны.
Тупой угол — измерение больше 90 градусов, но меньше 180 градусов.
Точка находится внутри угла, если она не лежит на самом угле и лежит на сегменте, конечные точки которого находятся по сторонам угла Биссектриса угла
Дополнительные углы — Если сумма двух углов равна 180 градусов.Каждый угол называется дополнением.
Дополнительные углы — Если сумма двух углов составляет 90 градусов. Каждый угол называется дополнительным.
Смежные углы — это два компланарных угла, которые имеют общую сторону, но не имеют общих внутренних точек.
Линейная пара — если два угла являются смежными и дополнительными.
Вертикальные углы — несмежные углы, образованные двумя пересекающимися линиями.
Вертикальные углы совпадают.
Все углы, имеющие одну и ту же вершину вокруг точки, называются углами в точке.
Свойства угла:
* Вертикальные углы равны
* Сумма углов в точке равна 360 градусам.
Поперечный — линия, которая пересекает две копланарные линии в двух разных точках.
Альтернативные внешние углы
— альтернативные внешние углы совпадают.
Альтернативные внутренние углы.
— пара несмежных внутренних углов на противоположных сторонах трансверсали.
— чередующиеся внутренние углы совпадают
Внутренние углы на одной стороне
— внутренние углы на одной стороне поперечной.
— Односторонние внутренние углы являются дополнительными
Соответствующие углы
— пара несмежных внутренних и внешних углов на одной стороне трансверсали.
— соответствующие углы конгруэнтны
Косые линии — линии, которые не компланарны и не пересекаются
Параллельные прямые — прямые, которые компланарны и не пересекаются
Наклонные прямые — линии, которые пересекают
Многоугольник — это плоская фигура, образованная тремя или больше сегментов, так что каждый сегмент пересекает ровно два других, по одному в каждой конечной точке, и никакие два сегмента с общей конечной точкой не лежат на одной прямой.
Последовательные стороны — две стороны многоугольника, имеющие общую конечную точку.
Последовательные углы — два угла, вершины которых являются конечными точками одной стороны.
Общая сторона двух последовательных углов называется включенной стороной двух углов.
Угол, содержащий общую вершину двух последовательных сторон, называется включенным углом двух сторон.
Отрезок, соединяющий любые две непоследовательные вершины, называется диагональю многоугольника. Многоугольник с совпадающими сторонами и равными углами — это правильный многоугольник.
Вогнутый — многоугольные области, изгибающиеся внутрь.
Convex — многоугольные участки, не загибающиеся внутрь.
Вогнутый многоугольник — многоугольник, определяющий вогнутую область.
Выпуклый многоугольник — многоугольник, определяющий выпуклую область.

Треугольник — многоугольник с 3 сторонами
* по сторонам
Равносторонний треугольник — имеет три равные стороны и три равных угла. Каждый угол равен 60 °.
Равнобедренный — имеет две равные стороны и два равных угла.
Чешуйчатый треугольник — не имеет совпадающих сторон.Другими словами, каждая сторона должна иметь разную длину.
* в соответствии с углами
Острый треугольник — имеет три острых угла (острый угол меньше 90 °)
Тупой треугольник — имеет тупой угол (тупой угол больше чем 90 °).
Прямой треугольник — имеет один прямой угол.

Параллельные линии — (сегмент или лучи) — это линии, которые пересекаются в одной точке.
Медиана треугольника — отрезок, концы которого являются вершиной треугольника и серединой противоположной стороны.
Центроид — точка совпадения трех медиан
треугольника.
Биссектриса угла треугольника — отрезок, делящий пополам угол треугольника, концы которого являются вершиной и точкой на противоположной стороне.

Incenter — точка совпадения трех биссектрис угла
треугольника.
Высота треугольника — отрезок от вершины треугольника перпендикулярно линии, содержащей противоположную сторону.
Ортоцентр — точка совпадения трех высот треугольника.
Серединный перпендикуляр стороны треугольника — прямая, точки которой равноудалены от концов данной стороны.
Circumcenter — точка совпадения
трех перпендикулярных биссектрис сторон треугольника.

Четырехугольники — четырехугольник — это многоугольник с четырьмя сторонами.
Параллелограмм — четырехугольник, у которого обе пары противоположных сторон параллельны.
Ромб — параллелограмм с четырьмя равными сторонами.
Прямоугольник — параллелограмм с четырьмя прямыми углами.
Квадрат — параллелограмм с четырьмя равными сторонами и четырьмя прямыми углами.
Воздушный змей — четырехугольник, у которого две пары смежных сторон совпадают, и никакие противоположные стороны не совпадают.
Трапеция — четырехугольник с одной парой параллельных сторон.

Сумма углов треугольника — Сумма внутренних углов треугольника в градусах равна 180.
Третье совпадение углов — Если два угла одного треугольника совпадают с двумя углами другого, то третьи углы равны конгруэнтный.
Угол, образующий прямую с углом треугольника, называется внешним углом.
Удаленные внутренние углы — это два угла в треугольнике, вершина которых не совпадает с вершиной внешнего угла.
Внешний угол треугольника (EAT) — Мера внешнего угла треугольника равна сумме измерений его двух удаленных внутренних углов.

Углы в многоугольнике
Имя многоугольника — количество сторон
Треугольник 3
Четырехугольник 4
Пятиугольник 5
Шестиугольник 6
Гептагон 7
Восьмиугольник 8
двенадцатигранник 12
Сумма углов выпуклого четырехугольника составляет 360 градусов .
Сумма внутренних углов многоугольника — сумма углов выпуклого многоугольника с n сторонами равна (n-2) 180.
Сумма размеров внешних углов по одному в каждой вершине любого выпуклого многоугольника равна 360.
Окружности и связанные с ними термины
Окружность — все точки находятся на одинаковом расстоянии от центра окружности.
Диаметр — хорда, содержащая центр окружности.
Длина диаметра в два раза больше длины радиуса.
Радиус — половина диаметра.
Диаметр делит окружность на две совпадающие дуги, называемые полукругами.
Две окружности совпадают, если их радиусы равны. Круг измеряет 360 градусов.
Окружность разделяет плоскость на три набора точек: сам круг, внутреннюю часть круга (набор точек на плоскости круга, расстояние от центра которых больше радиуса) и внешнюю часть круга.
Радиус
Периметр круга называется окружностью.
Дуги — две различные точки на окружности делят окружность на две части.
Полукруг — когда концы дуг также являются концами диаметра, каждая дуга представляет собой полукруг.В противном случае Minor arc — образуются большие дуги. Малая дуга меньше полукруга.
Центральный угол — это угол в плоскости окружности, вершиной которого является центр окружности.
Малая дуга — ее конечные точки лежат по бокам угла, а все остальные точки лежат внутри угла.
Большая дуга — точки на внешней стороне.
Вписанный угол — это угол, вершина которого лежит на окружности, а стороны содержат точки окружности.
Углы в одном сегменте — это вписанные углы, имеющие одинаковую пересеченную дугу.

Источник: E-Math (G7 Math Book), книжный магазин Rex

# MathWizard

Число точек, линий и плоскостей

Обзор

Вероятно, будет хорошей идеей уточнить, что мы подразумеваем под термином инцидентность в контексте геометрии. При чтении других страниц в этом разделе или при чтении других текстов, посвященных темам, связанным с геометрией, вы могли встретить термин совпадение .Две прямые называются совпадающими с , например, если они обе проходят через одни и те же две точки. Фактически это означает, что это одна и та же линия. Если два круга имеют радиусы разной длины, но один и тот же центр, их центры считаются совпадающими . Если две плоскости имеют три общие точки, которые не лежат на прямой линии, то они совпадают, то есть одна и та же плоскость (помните, что плоскость — это плоская двумерная поверхность, которая простирается до бесконечности в обоих направлениях вдоль каждой оси. ).

Что же тогда такое заболеваемость ? Проще говоря, это частичное совпадение . Две линии, пересекающиеся в одной точке, не совпадают, но у них есть общая точка. Из-за этого они, как говорят, инцидент друг к другу. Линия, проходящая через точку, называется , переходящей в эту точку . Линия, которая проходит через плоскость в одной точке, называется , падающей на эту плоскость на .И плоскость, которая пересекает другую плоскость, называется , падающей на на эту другую плоскость (как вы, вероятно, понимаете, пересечение двух плоскостей происходит вдоль линии , а не в одной точке). Заболеваемость подразумевается такими утверждениями, как «точка P лежит на прямой l» и «прямая a пересекает линию b» и так далее.

Заголовок этой страницы — «Количество точек, линий и плоскостей». Поэтому вы могли спросить себя, будет ли самый тривиальный случай инцидента включать в себя два момента, одно происшествие над другим.Однако это обязательно будет включать две точки, имеющие одинаковые координаты x и y (в случае двумерной среды) или одинаковые координаты x , y и z (в случай трехмерной среды). Если бы это было так, две точки по определению были бы совпадающими, а не случайными. Таким образом, падение может происходить между точкой и линией, между точкой и плоскостью, между двумя линиями, между линией и плоскостью или между двумя плоскостями, но не между двумя точками.

Существует ряд аксиом (называемых аксиомами инцидентности ), относящихся к инцидентности точек, линий и плоскостей. Аксиома — это утверждение или предложение, которое принимается как самоочевидно истинное, не требующее математического доказательства, и поэтому может использоваться в качестве отправной точки, из которой могут быть выведены другие утверждения или предложения. Аксиомы инцидентности бывают двух основных видов. Первая группа аксиом, которую мы рассмотрим, касается отношений между точками и линиями на плоскости (т.е., в двумерном пространстве). Вторая группа занимается отношениями между точками, линиями и плоскостями в трехмерном пространстве. В следующих разделах мы попытаемся объяснить значение каждой из этих аксиом.

Отношения инцидентов в самолете

Плоская геометрия связана с объектами, которые имеют не более двух измерений (например, сама плоскость и любые двумерные формы, существующие на этой плоскости).Точки безразмерны, , поскольку, хотя они определены фиксированными координатами x и y на плоскости относительно некоторой исходной точки, сами по себе они не имеют физического существования. Линии считаются одномерными , поскольку, хотя линия не имеет реальной субстанции (можно считать, что она состоит из бесконечного числа точек), она имеет длину не менее . Фактически, по определению, длина строки бесконечна.Плоскость — это плоская двухмерная поверхность .

В самом широком смысле термин геометрия инцидентности относится к области математики, которая включает изучение отношений инцидентности, которые могут существовать между точками, линиями, плоскостями и другими геометрическими объектами. В более узком смысле этот термин часто используется для описания конечной плоской геометрии (т.е. той, которая определяется конечным числом точек и линий). Мы будем интересоваться здесь только идеей евклидовой плоскости (т.е. тот, который простирается до бесконечности в каждом из двух своих измерений). Основные аксиомы инцидентности плоской геометрии изложены ниже.

  • Для любых двух различных точек на плоскости существует одна и только одна линия, которая будет проходить через обе точки

Линия A — единственная линия на плоскости, которая проходит через точку A и точку B .


На приведенном выше рисунке линия a проходит через точку A и точку через точку B.Линия a уникальна, потому что никакая другая линия не проходит через обе эти точки. Можно сказать, что линия пересекается с как с точкой A, так и с точкой B. Фактически мы могли бы взять любую пару различных точек из , лежащих на линии a, и использовать их для однозначного определения линии. Это дает безграничные возможности, поскольку по определению линия имеет бесконечную длину и, следовательно, на линии a находится бесконечное количество точек.

  • Если две прямые на плоскости пересекаются, они пересекаются ровно в одной точке и только в одной точке

Прямая a и линия b пересекаются только в точке P


Две прямые на одной плоскости, которые не пересекаются, по определению являются параллелью (т.е. в любой точке по длине кратчайшее расстояние между ними будет одинаковым). И наоборот, если две прямые в одной плоскости на , а не на параллельны, то они будут пересекаться в одной точке на плоскости. Эта точка будет единственной точкой, с которой соприкасаются обе линии.

  • Для линии a и точки P, которая не пересекается с точкой a, будет ровно одна линия и только одна линия, которые инцидентны точке P, но не пересекают линию a

Линия b — единственная линия, которая инцидентна точке P, но не пересекает линию a.


На приведенном выше рисунке линия b проходит через (т.е. инцидентно точке P, но не пересекает прямую a, потому что она параллельна ей. Никакая другая линия, лежащая в той же плоскости, что и линия a, не может проходить через точку P, не пересекая линию a. По определению, две параллельные прямые на евклидовой плоскости никогда не пересекаются. Однако изучение плоской геометрии может быть расширено, чтобы учесть возможность двух параллельных линий, которые в конечном итоге встречаются на бесконечности. Двумерная плоскость, допускающая такую ​​возможность, называется проективной плоскостью .Хотя мы не занимаемся здесь изучением проективной плоскости, интересно отметить, что идея параллельных линий, пересекающихся в бесконечности, как полагают, возникла в эпоху Возрождения, когда художники разрабатывали методы добавления перспективы к своим рисункам (в основном чтобы принять во внимание вид визуального эффекта, из-за которого кажется, что железнодорожные пути сходятся в некоторой воспринимаемой «точке схода»).


Параллельные линии сходятся в общей «точке схода».


Отношения инцидентности в трехмерном пространстве

Трехмерное пространство также содержит точки и линии.В отличие от двухмерного пространства (такого как наша евклидова плоскость), трехмерное пространство также может содержать множество плоскостей. Фактически, плоскость в трехмерном пространстве аналогична линии в двумерном пространстве в том смысле, что две плоскости либо параллельны друг другу, либо пересекаются. С другой стороны, прямые могут быть непараллельными, но не пересекать друг друга, и как прямые, так и точки могут совпадать с множеством различных (пересекающихся) плоскостей одновременно. Как вы, наверное, догадались, существует значительно большее количество возможностей, когда дело доходит до отношений инцидентности в трехмерном пространстве.Однако есть некоторые основные аксиомы, которые можно принять для трехмерного пространства:

  • Для любых двух различных точек в пространстве существует одна и только одна линия, которая будет проходить через обе точки (по сути, та же ситуация, которая существует для двух точек на плоскости)

Существует только одна линия, которая пересекает точку A и точку B.


На диаграмме выше точка A имеет координаты x = -2, y = -2, z = 3.Точка B имеет координаты x = 2, y = 2, z = -2. Линия l — единственная линия, которая проходит через обе точки. В двух измерениях линия должна полностью лежать внутри плоскости, чтобы существовать. Это означает, что все точки, составляющие линию, также полностью лежат в плоскости. Это также означает, что любая линия, проходящая через две точки на плоскости, также должна полностью лежать внутри плоскости. Очевидно, что отношения между линиями и плоскостями в трехмерном пространстве будут более сложными.Фактически, есть три возможности, относящиеся к линиям в трехмерном пространстве:

  • линия полностью лежит в плоскости
  • линия падает на самолет в единственной точке
  • прямая и плоскость не пересекаются (параллельны)

На предыдущем рисунке показано трехмерное пространство, определяемое осями x , y и z .По определению, любая пара осей будет пересекать друг друга под прямым углом, и будет одна плоскость, содержащая обе оси. Плоскость, содержащая оси x и y , изображена на рисунке в виде сетки. Прямая l падает на плоскость в одной точке. Ось y также пересекается с плоскостью и с каждой из двух других осей в одной точке. Он также перпендикулярен плоскости и двум другим осям. Таким образом, мы можем видеть два примера прямых, лежащих внутри плоскости (оси x и y ), и два примера прямых, пересекающих плоскость в одной точке (ось z и прямая l).На схеме ниже показана та же плоскость, что и раньше. Эта временная точка A имеет координаты x = -2, y = -1, z = 3, а точка B имеет координаты x = 2, y = 1, z = 3 Прямая l проходит через обе точки и параллельна плоскости в силу того, что координаты z для обеих точек одинаковы.


Линия l не пересекает плоскость, содержащую оси x и y .


Что еще можно сказать о линиях и плоскостях в трехмерном пространстве? Можно сказать, что если две прямые пересекаются, они инцидентны ровно в одной точке.Это верно как в двух, так и в трех измерениях. Мы также можем сказать, что существует одна плоскость, а только одна плоскость, которая содержит обе прямые. Если две линии пересекаются не , а , есть две возможности:

  • обе прямые лежат в одной плоскости и параллельны
  • ни одна плоскость не содержит обеих линий (линии считаются перекосом )

В приведенном выше примере линия l параллельна плоскости, содержащей оси x и y , но наклонена по отношению к самим осям x и y .Однако будет много прямых, содержащихся в плоскости, параллельных прямой l. На рисунке ниже точки A и B имеют те же координаты, что и ранее. Точка C имеет координаты x = -2, y = -1, z = 0, а точка B имеет координаты x = 2, y = 1, z = 0. Линия m проходит через точки C и D и полностью лежит в плоскости, содержащей оси x и y .Обратите внимание, что точки C и D имеют те же координаты x и y , что и точки A и B соответственно. Фактически, для любой точки на прямой m будет соответствующая точка на прямой l, которая имеет те же координаты x и y . Таким образом, линия m будет параллельна прямой l. Из этого мы можем сделать вывод, что существует плоскость, которая содержит как прямую l, так и прямую m (эта плоскость фактически будет перпендикулярна плоскости, содержащей оси x и y , и пересекает ее по линии m).


Линия l параллельна линии m


Есть еще несколько вещей, которые вы должны знать о взаимосвязи между линиями, точками и плоскостями в трехмерном пространстве.

  • для любых трех точек, которые не все лежат на одной прямой, существует одна (и только одна) плоскость, содержащая все три точки
  • для любой линии и некоторой точки, которая не лежит на этой линии, существует одна (и только одна) плоскость, которая содержит как линию, так и точку
  • для любых четырех точек, которые не все лежат на одной плоскости, не существует одной линии, содержащей более двух точек

Первая из этих аксиом в значительной степени подразумевает вторую, поскольку для любых двух точек будет уникальная линия, проходящая через обе из них.Логику, лежащую в основе последней из трех аксиом, не так-то легко понять. Иллюстрация ниже может помочь показать, как это работает. Все точки A, B и C лежат в плоскости, содержащей оси x и y . Пункт P — нет. Были построены прямые, содержащие каждую возможную пару из двух точек, и ни одна из линий не инцидентна более чем с двумя точками. Обратите внимание, что если бы точку C переместить так, чтобы она находилась на той же прямой, что и точки A и B (в попытке доказать, что правило неверно), на самом деле существовала бы плоскость, содержащая все четыре точки, поскольку мы бы имели эффективно уменьшил геометрию до той, которая содержит линию и точку не на этой линии.


Ни одна линия не связана более чем с двумя точками


В трехмерном пространстве, очевидно, может быть любое количество плоскостей (фактически, бесконечное число). Отношения между плоскостями в пространстве аналогичны отношениям между линиями на плоскости. Плоскости будут либо пересекать друг друга, либо быть параллельны друг другу. Разница, как упоминалось ранее, заключается в том, что две линии пересекаются в одной точке, а две плоскости пересекаются вдоль линии, как показано ниже.Показанные (конечные) плоскости перпендикулярны друг другу. Однако обратите внимание, что теоретически любое количество плоскостей может пересекать друг друга по одной и той же линии.


Две плоскости пересекаются по линии l


Для двух евклидовых плоскостей в космосе есть только две возможности. Плоскости либо пересекаются друг с другом по прямой, либо параллельны.Из этого мы можем сделать вывод, что если две плоскости имеют общую точку, эта точка также должна лежать на линии пересечения. И наоборот, каждая точка на линии пересечения является общей для обеих плоскостей. На следующем рисунке показаны три (конечные) плоскости, параллельные друг другу.


Три плоскости параллельно


Здесь стоит упомянуть еще две аксиомы, относящиеся к плоскостям, они описывают взаимосвязь между плоскостями и линиями, перпендикулярными им:

  • Две прямые, перпендикулярные одной плоскости, должны быть параллельны
  • Две плоскости, перпендикулярные одной линии, должны быть параллельны

Пересечения сферы

Пересечения сферы

Перекрестки со сферой

Каждое плоскостное пересечение сферы — это круг.Чтобы построить точки кривой пересечения сферы и другой поверхности, мы выбираем систему плоскостей, которые разрезают другую поверхность по простейшим возможным кривым.
Когда мы пересекаем сферу с поверхностью вращения, мы выбираем систему плоскостей, которые рассекают поверхность вращения по окружностям.
Пересечение двух сфер
Сфера пересекает плоскость на бесконечности по конике, которая называется абсолютной коникой пространства.

Кривая пересечения двух сфер всегда вырождается в абсолютную конику и окружность.

Следовательно, реальное пересечение двух сфер — это круг .
Плоскость, определяемая этим кругом, перпендикулярна линии, соединяющей центры сфер, и эта линия проходит через центр этого круга.

Пересечение сферы и цилиндра
Кривая пересечения сферы и цилиндра — это пространственная кривая 4-го порядка.
Эта кривая может быть кривой с одной ветвью в случае частичного пересечения, кривая с двумя ветвями в случае полного пересечения или кривая с одной двойной точкой, если поверхности имеют общую касательную плоскость.

ЧАСТИЧНОЕ ПЕРЕСЕЧЕНИЕ — кривая с одной ветвью

ПОЛНОЕ ПЕРЕСЕЧЕНИЕ — кривая с двумя ветвями

кривая пересечения с ОДНОЙ ДВОЙНОЙ ТОЧКОЙ

Если центр сферы лежит на оси цилиндра вращения, то кривая пересечения вырождается в две окружности.
Когда диаметр сферы равен диаметру основной окружности цилиндра, две окружности совпадают и две поверхности имеют одну и ту же касательную плоскость в каждой точке этой окружности.


два круга

один двойной круг

Пересечение сферы и конуса
Кривая пересечения сферы и конуса — это пространственная кривая 4-го порядка.
Эта кривая может быть кривой с одной ветвью в случае частичного пересечения, кривая с двумя ветвями в случае полного пересечения или кривая с одной двойной точкой ..
Кривая пересечения имеет двойную точку в двух случаях — если поверхности имеют общую касательную плоскость в регулярной точке или когда сфера проходит через двойную точку конуса, его вершину.

ЧАСТИЧНОЕ ПЕРЕСЕЧЕНИЕ — кривая с одной ветвью

ПОЛНОЕ ПЕРЕСЕЧЕНИЕ — кривая с двумя ветвями

кривая пересечения с ОДНОЙ ДВОЙНОЙ ТОЧКОЙ
Поверхности имеют общую касательную плоскость.

Leave A Comment